Как найти абсолютный угол преломления

Преломление света.

  • Закон преломления (частный случай).

  • Обратимость световых лучей.

  • Закон преломления (общий случай).

  • Полное внутреннее отражение.

  • Разберем задачи ЕГЭ по теме: Преломление света.

Автор — профессиональный репетитор, автор учебных пособий для подготовки к ЕГЭ Игорь Вячеславович Яковлев

Темы кодификатора ЕГЭ: закон преломления света, полное внутреннее отражение.

На границе раздела двух прозрачных сред наряду с отражением света наблюдается его преломление — свет, переходя в другую среду, меняет направление своего распространения.

Преломление светового луча происходит при его наклонном падении на поверхность раздела (правда, не всегда — читайте дальше про полное внутреннее отражение). Если же луч падает перпендикулярно поверхности, то преломления не будет — во второй среде луч сохранит своё направление и также пойдёт перпендикулярно поверхности.

к оглавлению ▴

Закон преломления (частный случай).

Мы начнём с частного случая, когда одна из сред является воздухом. Именно такая ситуация присутствует в подавляющем большинстве задач. Мы обсудим соответствующий частный случай закона преломления, а уж затем дадим самую общую его формулировку.

Предположим, что луч света, идущий в воздухе, наклонно падает на поверхность стекла, воды или какой-либо другой прозрачной среды. При переходе в среду луч преломляется, и его дальнейший ход показан на рис. 1.

Рис. 1. Преломление луча на границе «воздух–среда»

В точке падения O проведён перпендикуляр (или, как ещё говорят, нормаль) CD к поверхности среды. Луч AO, как и раньше, называется падающим лучом, а угол alpha между падающим лучом и нормалью — углом падения. Луч OB — это преломлённый луч; угол beta между преломлённым лучом и нормалью к поверхности называется углом преломления.

Всякая прозрачная среда характеризуется величиной n, которая называется показателем преломления этой среды. Показатели преломления различных сред можно найти в таблицах. Например, для стекла n=1,6, а для воды n=1,33. Вообще, у любой среды n textgreater 1; показатель преломления равен единице только в вакууме. У воздуха n=1,0003, поэтому для воздуха с достаточной точностью можно полагать в задачах n=1 (в оптике воздух не сильно отличается от вакуума).

Закон преломления (переход «воздух–среда»).

1) Падающий луч, преломлённый луч и нормаль к поверхности, проведённая в точке падения, лежат в одной плоскости.
2) Отношение синуса угла падения к синусу угла преломления равно показателю преломления среды:

frac{displaystyle sinalpha }{displaystyle sinbeta }=n. (1)

Поскольку n textgreater 1 из соотношения (1) следует, что sinalpha textgreater sinbeta , то есть alpha textgreater beta — угол преломления меньше угла падения. Запоминаем: переходя из воздуха в среду, луч после преломления идёт ближе к нормали.

Показатель преломления непосредственно связан со скоростью v распространения света в данной среде. Эта скорость всегда меньше скорости света в вакууме: v textless c. И вот оказывается,что

n=frac{displaystyle c }{displaystyle v }. (2)

Почему так получается, мы с вами поймём при изучении волновой оптики. А пока скомбинируем формулы . (1) и (2):

frac{displaystyle sinalpha }{displaystyle sinbeta }=frac{displaystyle c }{displaystyle v }. (3)

Так как показатель преломления воздуха очень близок единице, мы можем считать, что скорость света в воздухе примерно равна скорости света в вакууме c. Приняв это во внимание и глядя на формулу . (3), делаем вывод: отношение синуса угла падения к синусу угла преломления равно отношению скорости света в воздухе к скорости света в среде.

к оглавлению ▴

Обратимость световых лучей.

Теперь рассмотрим обратный ход луча: его преломление при переходе из среды в воздух. Здесь нам окажет помощь следующий полезный принцип.

Принцип обратимости световых лучей. Траектория луча не зависит от того, в прямом или обратном направлении распространяется луч. Двигаясь в обратном направлении, луч пойдёт в точности по тому же пути, что и в прямом направлении.

Согласно принципу обратимости, при переходе из среды в воздух луч пойдёт по той же самой траектории, что и при соответствующем переходе из воздуха в среду (рис. 2) Единственное отличие рис. 2 от рис. 1 состоит в том, что направление луча поменялось на противоположное.

Рис. 2. Преломление луча на границе «среда–воздух»

Раз геометрическая картинка не изменилась, той же самой останется и формула (1): отношение синуса угла alpha к синусу угла beta по-прежнему равно показателю преломления среды. Правда, теперь углы поменялись ролями: угол beta стал углом падения, а угол alpha — углом преломления.

В любом случае, как бы ни шёл луч — из воздуха в среду или из среды в воздух — работает следующее простое правило. Берём два угла — угол падения и угол преломления; отношение синуса большего угла к синусу меньшего угла равно показателю преломления среды.

Теперь мы целиком подготовлены для того, чтобы обсудить закон преломления в самом общем случае.

к оглавлению ▴

Закон преломления (общий случай).

Пусть свет переходит из среды 1 с показателем преломления n_{displaystyle 1} в среду 2 с показателем преломления n_{displaystyle 2}. Среда с большим показателем преломления называется оптически более плотной; соответственно, среда с меньшим показателем преломления называется оптически менее плотной.

Переходя из оптически менее плотной среды в оптически более плотную, световой луч после преломления идёт ближе к нормали (рис. 3). В этом случае угол падения больше угла преломления: alpha textgreater beta .

Наоборот, переходя из оптически более плотной среды в оптически менее плотную, луч отклоняется дальше от нормали (рис. 4). Здесь угол падения меньше угла преломления: alpha textless beta

Оказывается, оба этих случая охватываются одной формулой — общим законом преломления, справедливым для любых двух прозрачных сред.

Закон преломления.
1) Падающий луч, преломлённый луч и нормаль к поверхности раздела сред, проведённая в точке падения, лежат в одной плоскости.
2) Отношение синуса угла падения к синусу угла преломления равно отношению показателя преломления второй среды к показателю преломления первой среды:

frac{displaystyle sinalpha }{displaystyle sinbeta }=frac{displaystyle n_{displaystyle 2}}{displaystyle n_{displaystyle 1}}. (4)

Нетрудно видеть, что сформулированный ранее закон преломления для перехода «воздух–среда» является частным случаем данного закона. В самом деле, полагая в формуле (4) n_{displaystyle 1}=1, n_{displaystyle 2}=n, мы придём к формуле (1).

Вспомним теперь, что показатель преломления — это отношение скорости света в вакууме к скорости света в данной среде: n_{displaystyle 1}=c/v_{displaystyle1}, n_{displaystyle 2}=c/v_{displaystyle2}. Подставляя это в (4), получим:

frac{displaystyle sinalpha}{displaystyle sinbeta }=frac{displaystyle v_{displaystyle 1}}{displaystyle v_{displaystyle 2}}. (5)

Формула (5) естественным образом обобщает формулу (3). Отношение синуса угла падения к синусу угла преломления равно отношению скорости света в первой среде к скорости света во второй среде.

к оглавлению ▴

Полное внутреннее отражение.

При переходе световых лучей из оптически более плотной среды в оптически менее плотную наблюдается интересное явление — полное внутреннее отражение. Давайте разберёмся, что это такое.

Будем считать для определённости, что свет идёт из воды в воздух. Предположим, что в глубине водоёма находится точечный источник света S, испускающий лучи во все стороны. Мы рассмотрим некоторые из этих лучей (рис. 5).

Рис. 5. Полное внутреннее отражение

Луч SO_{displaystyle 1} падает на поверхность воды под наименьшим углом. Этот луч частично преломляется (луч O_{displaystyle 1} A_{displaystyle 1}) и частично отражается назад в воду (луч O_{displaystyle 1} B_{displaystyle 1}). Таким образом, часть энергии падающего луча передаётся преломлённому лучу, а оставшаяся часть энергии -отражённому лучу.

Угол падения луча S O_{displaystyle 2} больше. Этот луч также разделяется на два луча — преломлённый и отражённый. Но энергия исходного луча распределяется между ними по-другому: преломлённый луч O_{displaystyle 2} A_{displaystyle 2} будет тусклее, чем луч O_{displaystyle 1} A_{displaystyle 1} (то есть получит меньшую долю энергии), а отражённый луч O_{displaystyle 2} B_{displaystyle 2} — соответственно ярче, чем луч O_{displaystyle 1} B_{displaystyle 1} (он получит большую долю энергии).

По мере увеличения угла падения прослеживается та же закономерность: всё большая доля энергии падающего луча достаётся отражённому лучу, и всё меньшая — преломлённому лучу. Преломлённый луч становится всё тусклее и тусклее, и в какой-то момент исчезает совсем!

Это исчезновение происходит при достижении угла падения alpha _{0}, которому отвечает угол преломления 90^{circ}. В данной ситуации преломлённый луч OA должен был бы пойти параллельно поверхности воды, да идти уже нечему — вся энергия падающего луча SO целиком досталась отражённому лучу OB.

При дальнейшем увеличении угла падения преломлённый луч и подавно будет отсутствовать.

Описанное явление и есть полное внутреннее отражение. Вода не выпускает наружу лучи с углами падения, равными или превышающими некоторое значение alpha _{0} — все такие лучи целиком отражаются назад в воду. Угол alpha _{0} называется предельным углом полного отражения.

Величину alpha _{0} легко найти из закона преломления. Имеем:

frac{displaystyle sinalpha _{0}}{displaystyle sin90^{circ}}=frac{displaystyle 1}{displaystyle n}.

Но sin90^{circ}=1, поэтому

sinalpha _{0}=frac{displaystyle 1}{displaystyle n},

откуда

alpha _{0}=arcsinfrac{displaystyle 1}{displaystyle n}.

Так, для воды предельный угол полного отражения равен:

alpha _{0}=arcsinfrac{displaystyle 1}{1,33} approx 48,8^{circ} .

Явление полного внутреннего отражения вы легко можете наблюдать дома. Налейте воду в стакан, поднимите его и смотрите на поверхность воды чуть снизу сквозь стенку стакана. Вы увидите серебристый блеск поверхности — вследствие полного внутреннего отражения она ведёт себя подобно зеркалу.

Важнейшим техническим применением полного внутреннего отражения является волоконная оптика. Световые лучи, запущенные внутрь оптоволоконного кабеля (световода) почти параллельно его оси, падают на поверхность под большими углами и целиком, без потери энергии отражаются назад внутрь кабеля. Многократно отражаясь, лучи идут всё дальше и дальше, перенося энергию на значительное расстояние. Волоконно-оптическая связь применяется, например, в сетях кабельного телевидения и высокоскоростного доступа в Интернет.

к оглавлению ▴

Разберем задачи ЕГЭ по теме: Преломление света.


Задача 1. Нижняя грань AC прозрачного клина посеребрена и представляет собой плоское зеркало. Угол при вершине клина alpha = 30^{circ} . Луч света падает из воздуха на клин перпендикулярно грани AB, преломляется и выходит в воздух через ту же грань AB, но уэе под углом преломления beta = 90^{circ} Определите показатель преломления материала клина. Сделайте рисунок, поясняющий ход луча в клине.

Дано:
alpha = 30^{circ}
beta = 90^{circ}
n-?
Решение. Решение задач по геометрической оптике необходимо начинать с построения чертежа (рисунка), моделирующего условия, описанные в тексте задачи.

Световой луч падает на прозрачный клин перпендикулярно стороне АВ (см.рис.1). В этом случае, световой луч не преломляется на границе раздела воздух-клин, так как угол падения равен 0, соответственно, угол преломления также равен 0. Следовательно, внутри клина световой луч попадает на нижнюю грань АС, которая представляет собой плоское зеркало. Согласно рис.1 величина угла alpha_1=180^{circ}-(alpha+90^{circ})=90^{circ}-alpha.

alpha_1=90^{circ}-30^{circ}=60^{circ}.

Тогда угол падения луча на плоское зеркало будет равен
90^{circ}-alpha_1=90^{circ}-60^{circ}=30^{circ}.

То есть угол падения равен alpha=30^{circ}.
Согласно закону отражения света, угол падения светового луча равен углу отражения. В треугольнике МКО угол КОМ образован суммой двух углов α, поэтому он равен 60°. Тогда угол падения светового луча на грань АВ также будет равен 2alpha=60^{circ} (равенство накрест лежащих углов).
На следующем этапе задачи надо применить закон преломления света, так как луч переходит из одной среды в другую.
frac{sin{2alpha}}{sinbeta}=frac{1}{n}
При записи этой формулы учтено, что второй средой является воздух с показателем преломления равным 1, а первой средой является материал клина с показателем преломления n, который необходимо определить. Из последней формулы можно выразить и рассчитать n.

n=frac{sinbeta}{sin2alpha}

n=frac{sin90^{circ}}{sin(2cdot 30^{circ})}=frac{sin90^{circ}}{sin60^{circ}}approx 1,15

Ответ: 1,15

Задача 2. На тонкую собирающую линзу от удалённого источника падает пучок параллельных лучей (см. рисунок). Как изменится положение изображения источника, создаваемого линзой, если между линзой и её фокусом поставить
плоскопараллельную стеклянную пластинку с показателем преломления n (на рисунке положение пластинки отмечено пунктиром)? Ответ поясните, указав, какие физические закономерности Вы использовали. Сделайте рисунок, поясняющий ход лучей до и после установки плоскопараллельной стеклянной пластинки.

Решение. Рассмотрим ход световых лучей от удаленного источника через линзу при отсутствии плоскопараллельной стеклянной пластинки (см.рис.1).

Луч 1-1ʹ проходит через оптический центр линзы и не преломляется. Луч 2-2ʹ идет через фокус и после прохождения через линзу, идет параллельно главной оптической оси. Пересечение этих двух лучей дает действительное изображение удаленного источника, которое расположено в фокальной плоскости линзы. Этот факт также можно доказать, используя формулу тонкой линзы.

frac{1}{d}+frac{1}{f}=frac{1}{F} (1)

Так как источник света расположен на расстоянии d rightarrow infty, то frac{1}{d}rightarrow 0.

Тогда формула тонкой линзы (1) примет вид frac{1}{f}=frac{1}{F}, следовательно, f=F, т.е. изображение формируется в фокальной плоскости линзы.

Рассмотрим ход световых лучей через плоскопараллельную стеклянную пластинку. Для этого необходимо использовать закон преломления света.

Рис.2

Согласно рис.2 угол падения луча на пластину равен α. Закон преломления света на границе раздела воздух-пластинка имеет вид:

frac{sinalpha}{sinbeta}=frac{n}{1}=n (1).

Здесь учтено, что показатель преломления воздуха равен 1, а пластинки n.
При переходе светового луча из пластинки в воздух, закон преломления света будет иметь вид:

frac{sinbeta}{singamma}=frac{1}{n} (2).

В этом случае первой средой является пластинка с показателем преломления n, а второй средой будет воздух с показателем преломления равным 1.
Из (1) и (2) выразим sinalpha и singamma.

sinalpha=nsinbeta и singamma=nsinbeta.

Так как правые части этих уравнений равны, то sinalpha=singamma.

Отсюда вытекает равенство углов alpha=gamma. Следовательно, луч, падающий на стеклянную пластину, выходит из нее, оставаясь параллельным входящему лучу. Но при этом выходящий луч немного смещается вверх.

Исходя из этого можно сделать вывод, что изображение удаленного источника после прохождения через плоскопараллельную стеклянную пластину, не изменится. Из удаленного источника выходит бесконечное количество параллельных лучей, которые собираются в фокальной плоскости линзы.

Ответ: не изменится.


Задача 3. Ученик провел опыт по преломлению света, представленный на фотографии. Как изменится при уменьшении угла падения угол преломления светового пучка и скорость света, распространяющегося в стекле? Для каждой величины определите соответствующий характер изменения:
1) увеличится
2) уменьшиться
3) не изменится
Запишите в таблицу выбранные цифры для каждой величины. Цифры в ответе могут повторяться.

Угол преломления Скорость света в стекле

Решение. Для ответа на первый вопрос задачи необходимо применить закон преломления света для границы раздела воздух-стекло.

frac{sinalpha}{sinbeta}=frac{n}{1}=n.

Показатель преломления стекла равен n, а воздуха 1.
При уменьшении угла падения α, будет уменьшаться и значение sinalpha. Так как показатель преломления стекла не изменяется, то значение sinbeta так же будет уменьшаться. Поэтому угол преломления уменьшится.

Для ответа на второй вопрос надо учесть, что скорость света в данной среде определяется значением показателя преломления v_{cp}=frac{c}{n}, где с – скорость света в вакууме, а n – показатель преломления среды (стекла). Так как эти обе величины не изменяются, то скорость света в стекле так же не изменяется.

Ответ: 23.

Задача 4. Чему равен синус предельного угла полного внутреннего отражения при переходе света из вещества с n_1=1,5 в вещество с n_2=1,2?

Решение.

Явление полного внутреннего отражения наблюдается при переходе светового луча из оптически более плотной среды в оптически менее плотную (см.рис.1). Источник света S должен находиться в среде с большим показателем преломления.

Для нахождения синуса угла полного внутреннего отражения необходимо воспользоваться законом преломления света.

frac{sinalpha_{np}}{sinbeta}=frac{n_2}{n_1} (1)

При полном внутреннем отражении преломленный луч скользит по границе раздела двух сред и угол преломления beta=90^{circ}. С учетом того, что sin90^{circ}=1 уравнение (1) примет вид:

sinalpha_{np}=frac{1,2}{1,5}=0,8

sinalpha_{np}=0,8

Ответ: 0,8.

Если вам нравятся наши материалы — записывайтесь на курсы подготовки к ЕГЭ по физике онлайн

Благодарим за то, что пользуйтесь нашими публикациями.
Информация на странице «Преломление света.» подготовлена нашими редакторами специально, чтобы помочь вам в освоении предмета и подготовке к ЕГЭ и ОГЭ.
Чтобы успешно сдать необходимые и поступить в ВУЗ или техникум нужно использовать все инструменты: учеба, контрольные, олимпиады, онлайн-лекции, видеоуроки, сборники заданий.
Также вы можете воспользоваться другими статьями из данного раздела.

Публикация обновлена:
08.05.2023

Из прошлых уроков вы уже знаете, что в однородной среде свет распространяется прямолинейно. Но в жизни много ситуаций, когда свет проходит через разные вещества до того, как достигнет наших глаз.

Например, через оконные стекла мы отлично видим все, что происходит на улице. А через стекла в межкомнатных дверях мы можем видеть только размытые силуэты того, что находится за дверью. Тот же самый пример можно привести и с прозрачной и мутной водой. 

Значит, получаемое нашими глазами изображение как-то связано с тем, через какие среды проходит свет. Двигаясь прямолинейно в одной среде, он переходит в другую и снова двигается прямолинейно. Что же происходит при этом переходе из одной среды в другую? 

Так, вам предстоит узнать новое понятие — преломление света. В ходе данного урока вы узнаете закономерности этого явления, рассмотрите различные опыты и научитесь применять полученные знания для решения задач. 

Явление преломления света

Рассмотрим простой опыт. Для него нам понадобится прозрачный стакан с водой и обычный карандаш (рисунок 1).

Рисунок 1. Демонстрация преломления света

Сначала опустим карандаш в воду вертикально (рисунок 1, а). Части карандаша в воздухе и в воде не изменились.

А теперь поменяем угол наклона карандаша (рисунок 2, б). Мы увидим интересную картинку. Нам кажется, что карандаш переломился на границе воды и воздуха.

Что произошло? Мы видим карандаш, потому что на него падает свет от какого-то источника. Его лучи отражаются от карандаша и попадают нам в глаза. Когда мы опустили карандаш в воду под каким-то углом, световые лучи дошли до наших глаз не только через воздух, но еще и через воду в стакане. При этом они поменяли направление своего распространения при переходе из одной среды в другую. В таком случае говорят, что свет преломился.

Преломление света — это явление изменения направления распространения света при переходе из одной среды в другую.

Но, если свет преломляется при переходе из одной среды в другую, почему на рисунке 1 (а) мы все равно видим карандаш без изменений? Чтобы разобраться с этим вопросом, нам необходимо более подробно изучить природу преломления света.

Скорость света и оптическая плотность среды

Свет распространяется в пространстве с определенной скоростью. Эта скорость настолько велика, что нам кажется, будто свет появляется мгновенно. Например, когда в темной комнате мы щелкаем переключателем, и включается свет.

Ученые не только рассчитали значение этой скорости, но и доказали, что скорость света различается в разных средах (таблица 1).

Вещество $c$, $frac{км}{с}$
Воздух 300 000
Вода 225 000
Стекло 198 000
Сероуглерод 184 000
Алмаз 124 000
Таблица 1. Значения скорости света в различных средах

Значения скорости света в вакууме и воздухе практически не отличаются, поэтому используют одно значение — $300 000 frac{км}{с}$. Эта величина обозначается буквой $c$. 

В других же средах наблюдается значительная разница в значениях скорости. Например, в воде скорость света меньше, чем в воздухе. При этом говорят, что вода является оптически более плотной средой, чем воздух.

Оптическая плотность — это величина, которая характеризует различные среды в зависимости от значения скорости распространения света в них.

Если пучок света падает на поверхность, разделяющую две прозрачные среды с разной оптической плотностью, то часть света отразится от этой поверхности, а другая часть проникнет во вторую среду. При этом луч света изменит свое направление — происходит преломление света.

Схема преломления светового луча. Угол преломления

Рассмотрим преломление света более подробно (рисунок 2).

Рисунок 2. Схема преломления светового луча при переходе из воздуха в воду

Перечислим элементы, обозначенные на рисунке 2:

  • MN — граница раздела воздуха и воды
  • Луч AO — падающий луч
  • Луч OB — преломленный луч
  • CD — перпендикуляр, опущенный к поверхности раздела двух сред и проведенный через точку падения O
  • Угол AOC — угол падения ($alpha$)
  • Угол DOB — угол преломления ($gamma$)

Угол преломления — это угол между перпендикуляром, опущенным к границе раздела двух сред в точке падения светового луча, и преломленным лучом.

Направления луча при переходе в воду изменилось. Луч света стал ближе к перпендикуляру CD. Т.е., $gamma < alpha$. Рассмотрим опыт, который нам наглядно демонстрирует этот факт.

Возьмем стеклянный сосуд и наполним его водой. Воду подкрасим флуоресцентной жидкостью. Она будет светится в тех местах, где на нее будет попадать яркий свет — это удобно для наших наблюдений. На дно сосуда поместим плоское зеркало (рисунок 3).

Рисунок 3. Преломление света на опыте с плоским зеркалом

Теперь на поверхность воды с помощью маленького фонарика направим пучок света. Сделаем это таким образом, чтобы пучок света падал под каким-то углом.

Мы увидим, как луч поменяет свое направление на границе воздуха и воды. При этом угол преломления заметно меньше угла падения ($gamma_1 < alpha_1$). 

Далее луч отразится от плоского зеркала и снова достигнет границы раздела двух сред. Теперь мы видим, что луч падения заметно меньше луча преломления ($gamma_2 > alpha_2$). 

Вода — более плотная оптическая среда, чем воздух. Из всего этого мы можем сделать следующие выводы:

  1. Если свет идет из оптически менее плотной среды в более плотную, то угол преломления всегда меньше угла падения: $gamma < alpha$
  2. Если свет идет из оптически более плотной среды в менее плотную, то угол преломления всегда больше угла падения: $gamma > alpha$

Если в ходе опытов мы будем менять угол падения, то заметим, что угол преломления тоже будет изменяться. При этом вышеописанные нами закономерности будут исполняться.

Показатель преломления

Давайте выясним, как именно углы падения и преломления связаны друг с другом. Рассматривать будем луч света падающий из воздуха в воду.

При увеличении угла падения, будет увеличиваться угол преломления (рисунок 4). Но отношение между этими углами ($frac{alpha}{gamma}$) не будет постоянным.

Рисунок 4. Зависимость угла преломления от угла падения

Постоянным будет оставаться другое отношение этих углов — отношение их синусов:
$frac{sin 30 degree}{sin 23 degree} = frac{sin 45 degree}{sin 33 degree} = frac{sin 60 degree}{sin 42 degree} approx 1.33$.

Полученное число (1.3) называют относительным показателем преломления. Обозначают эту величину буквой $n_{21}$.

Так, для любой пары веществ с разными оптическими плотностями можно записать:

$frac{sin alpha}{sin gamma} = n_{21}$.

Чем больше относительный показатель преломления, тем сильнее преломляется световой луч при переходе из одной среды в другую.

В чем физический смысл этой величины? Ранее мы говорили, что оптическая плотность характеризует вещество по скорости распространения света в нем. Показатель преломления делает то же самое.

Относительный показатель преломления — это величина, показывающая, во сколько раз скорость света в первой по ходу луча среде отличается от скорости распространения света во второй среде:
$n_{21} = frac{upsilon_1}{upsilon_2}$.

Если луч света падает из вакуума или воздуха в какое-то вещество, то используется еще одна величина — абсолютный показатель преломления.

Абсолютный показатель преломления — это величина, показывающая во сколько раз скорость света в вакуумевоздухе больше, чем в данной среде:
$n = frac{c}{upsilon}$,
где $c = 3 cdot 10^8 frac{м}{с}$.

В таблице 2 представлены значения абсолютных показателей преломления некоторых веществ. Иногда их называют относительными показателями преломления относительно воздуха, потому что для воздуха $n = 1$.

Вещество $n$
Воздух 1.00
Лед 1.31
Вода 1.33
Спирт 1.36
Стекло (обычное) 1.50
Стекло (оптическое) 1.47 — 2.04
Рубин 1.76
Алмаз 2.42
Таблица 2. Абсолютные показатели преломления света различных веществ

Выразим относительный показатель преломления $n_{21}$ через абсолютные показатели преломления $n_1$ и $n_2$:
$n_{21} = frac{upsilon_1}{upsilon_2} = frac{frac{c}{n_1}}{frac{c}{n_2}} = frac{n_2}{n_1}$.

Относительный показатель преломления $n_{21}$ имеет нижний индекс $21$, который читается как: «два один». Этот индекс связан с полученной нами формулой: $n_{21} = frac{n_2}{n_1}$. То есть, относительный показатель преломления $n_{21}$ равен отношению абсолютных показателей $n_2$ к $n_1$. При этом нижние индексы обозначают последовательность сред, через которые проходит световой луч.

Здесь мы вернемся к вопросу о том, почему на рисунке 1 (а) мы не видим преломления.

Если падающий луч падает перпендикулярно на границу раздела двух сред, то он не испытывает преломления.

Доказывается это опытным путем. При любых других углах падения, отличных от $0 degree$, преломление света происходит по вышеописанным закономерностям.

Закон преломления света

Итак, преломление света происходит по определенному закону.

Закон преломления света:
падающий и преломленный лучи и перпендикуляр, проведенный к границе раздела двух сред в точке падения луча, лежат в одной плоскости. При этом отношение синуса угла падения к синусу угла преломления — постоянная величина для двух сред:
$frac{sin alpha}{sin gamma} = frac{n_2}{n_1} = n_{21}$.

Мнимое изображение, образованное преломлением света. Призмы

Преломление света, как и отражение света плоским зеркалом, создает “кажущееся” изменение положение источника света. Мы наблюдали такое изменение в самом первом опыте этого урока на рисунке 1, б.

Но, дело в том, что мнимое положение источника света в случае преломления будет различным для лучей, падающих на границу раздела двух сред под разными углами. Поэтому мнимое положение источника света при преломлении обычно подробно не рассматривают.

Тем не менее, мы часто замечаем эти изменения. Например, в прозрачной воде в закрытых водоемах или в море кажется, что предметы, лежащие на дне и находящиеся в толще воды, находятся на другом расстоянии от нас, чем они есть на самом деле.

Рассмотрим наглядный опыт с монеткой (рисунок 5).

Рисунок 5. Опыт с монеткой

Возьмем неглубокую широкую чашку и положим на ее дно монетку. Выберем такое положение для наблюдения, чтобы она была не видна (рисунок 5, а). 

Оставаясь в этой же точке наблюдения, нальем в чашку воду. Теперь монета стала видна (рисунок 5, б). То есть, мы видим не саму монету, а ее мнимое изображение, образованное преломлением света.

В различных оптических приборах используют эти особенности преломления. Часто свет проходит сквозь тело, имеющее форму призмы (рисунок 6, а).

Рисунок 6. Прохождение светового луча через призму

Световой луч, падающий на боковую грань призмы дважды преломляется (рисунок 6, б): при входе в призму и при выходе из нее. Такой луч на выходе из призмы будет отклоняться к основанию треугольника.

В оптических приборах используют не просто призмы, но и их различные сочетания. Например, на рисунке 7 изображены 3 коробки, в которых находятся треугольные призмы.

Рисунок 7. Различные положения призм, используемые для изменения хода световых лучей

Вы можете оценить, как при разных положениях призм изменяется ход лучей на выходе из коробки. При этом падающие лучи во всех трех случаях (а, б, в) были параллельны и имели одинаковое направление.

Примеры задач

Задача №1

Луч света переходит из скипидара в воздух. Определите абсолютный показатель преломления скипидара, если при угле падения, равном $30 degree$, угол преломления равен $45 degree$ (рисунок 8). Чему равна скорость распространения света в скипидаре?

Рисунок 8. Задача на преломление света №1

Дано:
$alpha = 30 degree$
$gamma = 45 degree$
$n_2 = 1$
$c = 3 cdot 10^8 frac{м}{с}$

$n_1 — ?$
$upsilon_1 — ?$

Посмотреть решение и ответ

Скрыть

Решение:

Так как световой луч проходит из скипидара (первая среда) в воздух (вторая среда), мы обозначили абсолютный показатель скипидара как $n_1$, а воздуха как $n_2$.

По закону преломления света:
$frac{sin alpha}{sin gamma} = n_{21} = frac{n_2}{n_1}$.

Для воздуха $n_2 = 1$, поэтому:
$frac{sin alpha}{sin gamma} = frac{1}{n_1}$.

Выразим $n_1$:
$n_1 = frac{sin gamma}{sin alpha}$.

Рассчитаем $n_1$:
$n_1 = frac{sin 45 degree}{sin 30 degree} = frac{frac{sqrt{2}}{2}}{frac{1}{2}} = sqrt{2} approx 1.41$.

По определению абсолютного показателя преломления для скипидара мы можем записать:
$n_1 = frac{c}{upsilon_1}$.

Выразим $upsilon_1$ и рассчитаем:
$upsilon_1 = frac{c}{n_1} = frac{3 cdot 10^8 frac{м}{с}}{1.41} approx 2 cdot 10^8 frac{м}{с}$.

Ответ: $n_1 approx 1.41$, $upsilon_1 approx 2 cdot 10^8 frac{м}{с}$.

Задача №2

Световой луч падает из воздуха в стекло. Абсолютный показатель преломления стекла равен $1.73$. Чему равен угол преломления, если отраженный луч образует с перпендикуляром, опущенным в точку падения луча на границе раздела двух сред, угол, равный $60 degree$?

При решении задачи мы будем использовать рисунок 9.

Рисунок 9. Задача на преломление света №2

$AO$ — падающий луч, а угол $alpha$ — угол падения. Луч $AO$ падает на границу раздела двух сред (воздуха и стекла). Образуются отраженный луч $OB$ и преломленный луч $OC$. Им соответствуют угол отражения $beta$ и угол преломления $gamma$.

Теперь запишем условие задачи и решим ее.

Дано:
$n_1 = 1$
$n_2 = 1.73$
$beta = 60 degree$

$gamma — ?$

Посмотреть решение и ответ

Скрыть

Решение:

По закону отражения света:
$alpha = beta = 60 degree$.

По закону преломления света:
$frac{sin alpha}{sin gamma} = frac{n_2}{n_1}$.

Для воздуха $n_1 = 1$, поэтому:
$frac{sin alpha}{sin gamma} = n_2$.

Выразим синус угла преломления и рассчитаем его:
$sin gamma = frac{sin alpha}{n_2} = frac{sin 60 degree}{1.73} = frac{frac{sqrt{3}}{2}}{1.73} = frac{sqrt{3}}{3.46} approx frac{1.73}{3.46} = 0.5 = frac{1}{2}$.

Если $sin gamma = frac{1}{2}$, то $gamma = 30 degree$.

Ответ: $gamma = 30 degree$.

Задача №3

На дне пруда глубиной $3 space м$ находится источник света. Показатель преломления воды равен $1.33$, а воздуха — $1$. На какой глубине наблюдатель увидит источник света, если он смотрит вертикально вниз с лодки.

Условие задачи дает понять, что в глаз наблюдателя попадает луч, который падает перпендикулярно границе раздела двух сред. В таком случае, преломление наблюдаться не будет. Тем не менее, как и в настоящей жизни, мы все равно увидим преломленное изображение источника света. Он будет казаться ближе. В ходе решения этой задачи вы узнаете, почему так происходит.

Для начала рассмотрим рисунок 10.

 Рисунок 10. Задача на преломление света №3

Источник света $S$ находится на глубине $H$. Мы опишем его двумя лучами: $SA$ и $SO$. Луч $SA$ перпендикулярен к границе раздела двух сред. Поэтому он не преломляется. Луч  $SO$ достигает границы раздела под некоторым углом. Он образует с перпендикуляром $CD$ угол падения $alpha$. Далее этот луч преломляется под углом преломления $gamma$ и попадает в глаза наблюдателя (точка $B$).

Продолжим преломленный луч до луча $SA$. Этот луч мы будем использовать как перпендикуляр к поверхности воды, чтобы оценивать глубину. Мы получили точку $S_1$ — мнимое изображение источника света. Соответственно длина отрезка $AS$ — это реальная глубина пруда $H$, а длина отрезка $AS_1$ — мнимая глубина $h$.

Обратите внимание, что мы взяли второй луч $SO$ не просто так — он падает под крайне малым углом $alpha$. После преломления мы получаем такой малый угол $gamma$, что он попадает в глаз наблюдателя. Т.е., на рисунке 8 схематическая область увеличена для нашего удобства во много раз. Мы рассматриваем настолько малые углы, что преломленный луч $SB$ достигает глаза, и мы видим мнимое изображение, образованное преломлением света.

Теперь мы можем записать условие задачи и решить ее.

Дано:
$H = 3 space м$
$n_1 = 1.33$
$n_2 = 1$

$h — ?$

Посмотреть решение и ответ

Скрыть

Решение:

Рассмотрим две прямые $AS$ и $CD$. Они параллельны, а прямая $SO$ — секущая. Тогда накрест лежащие углы равны друг другу:
$angle ASO = alpha$.

Запишем тангенс этого угла в прямоугольном треугольнике $ASO$:
$tg alpha = frac{AO}{AS} = frac{AO}{H}$.
Тогда, $AO = H cdot tg alpha$.

Теперь попробуем выразить $AO$ из другого треугольника — $AS_1O$.
Если рассмотрим $S_1O$ как прямую, пересекающую две параллельные прямые, то $angle AS_1O = gamma$.

Запишем тангенс этого угла:
$tg gamma = frac{AO}{AS_1} = frac{AO}{h}$.
Тогда, $AO = h cdot tg gamma$.

Получается, что $H cdot tg alpha = h cdot tg gamma$.
Выразим отсюда мнимую глубину $h$:
$h = H cdot frac{tg alpha}{tg gamma}$.

Так как углы $alpha$ и $gamma$ крайне малы, мы можем смело использовать следующие приближения:
$tg alpha approx sin alpha$,
$tg gamma approx sin gamma$.

Тогда, $h = H cdot frac{sin alpha}{sin gamma} = H cdot frac{n_2}{n_1}$.
Так как $n_2 = 1$, мы можем записать, что $h = frac{H}{n_1}$.

$h = frac{3 space м}{1.33} approx 2.3 space м$.

Ответ: $h = 2.3 space м$.

Содержание:

Преломление света:

Почему ложка, опущенная в стакан с водой, кажется нам сломанной на границе воздуха и воды? Что такое оптическая плотность среды? Как ведет себя свет, переходя из одной среды в другую? Обо всем этом вы узнаете из этого параграфа.

Опыты по преломлению света

Проведем такой эксперимент. Направим на поверхность воды в широком сосуде узкий пучок света под некоторым углом к поверхности. Мы заметим, что в точках падения лучи не только отражаются от поверхности воды, но и частично проходят в воду, изменяя при этом свое направление (рис. 3.33).

Изменение направления распространения света в случае его прохождения через границу раздела двух сред называют преломлением света.

Первое упоминание о преломлении света можно найти в работах древнегреческого философа Аристотеля, который задавался вопросом: почему палка в воде кажется сломанной? А в одном из древнегреческих трактатов описан такой опыт: «Нужно встать так, чтобы плоское кольцо, положенное на дно сосуда, спряталось за его краем.
Преломление света в физике - формулы и определения с примерами

Потом, не изменяя положения глаз, налить в сосуд воду. Луч света преломится на поверхности воды, и кольцо станет видимым». Аналогичный опыт проиллюстрирован на рис. 3.34.

Причина преломления света

Так почему же свет, переходя из одной среды в другую, изменяет свое направление?

Мы уже знаем, что свет в вакууме распространяется хотя и с огромной, но тем не менее конечной скоростью — около 300 000 км/с. В любой другой среде скорость света меньше, чем в вакууме.

Преломление света в физике - формулы и определения с примерами

Например, в воде скорость све-та в 1,33 раза меньше, чем в вакууме; когда свет переходит из воды в алмаз, его скорость уменьшается еще в 1,8 раза; в воздухе скорость распространения света в 2,4 раза больше, чем в алмазе, и лишь немного ( = 1,0003 раза) меньше скорости света в вакууме. Именно изменение скорости света в случае перехода из одной прозрачной среды в другую является причиной преломления света.

Принято говорить об оптической плотности среды: чем меньше скорость распространения света в среде, тем большей является оптическая плотность среды.

Так, воздух имеет большую оптическую плотность, чем вакуум, поскольку в воздухе скорость света несколько меньше, чем в вакууме. Оптическая плотность воды меньше, чем оптическая плотность алмаза, поскольку скорость света в воде больше, чем в алмазе.

Чем больше отличаются оптические плотности двух сред, тем более преломляется свет на границе их раздела. Другими словами, чем больше изменяется скорость света на границе раздела двух сред, тем сильнее он преломляется.

Закономерности преломления света

Рассмотрим явление преломления света подробнее. Для этого снова воспользуемся оптической шайбой. Установив в центре диска стеклянный полуцилиндр, направим на него узкий пучок света (рис. 3.35). Часть пучка отразится от поверхности полуцилиндра, а часть пройдет сквозь него, изменив свое направление (преломится).

На схеме по правую сторону луч SO задает направление падающего пучка света, луч ОК — направление отраженного пучка, луч ОВ — направление
Преломление света в физике - формулы и определения с примерами

Преломление света в физике - формулы и определения с примерами

Рис. 3.36. Установление закономерности преломления света Преломление света в физике - формулы и определения с примерами— углы падения, Преломление света в физике - формулы и определения с примерами— углы преломления).

В случае увеличения угла падения света увеличивается и угол его преломления. Если свет падает из среды с меньшей оптической плотностью в среду с большей оптической плотностью (из воздуха в стекло) (а), то угол падения больше угла преломления. Если наоборот (из стекла в воздух) (б), то угол преломления больше угла падения преломленного пучка; MN — перпендикуляр, восставленный в точке падения луча SO. Все указанные лучи лежат в одной плоскости — в плоскости поверхности диска.

Угол, образованный преломленным лучом и перпендикуляром к границе деления двух сред, восставленным в точке падения луча, называется углом преломления.

Если теперь увеличить угол падения, то мы увидим, что увеличится и угол преломления. Уменьшая угол падения, мы заметим уменьшение угла преломления (рис. 3.36).

Соотношение значений угла падения и угла преломления в случае перехода пучка света из одной среды в другую зависит от оптической плотности каждой из сред. Если, например, свет падает из воздуха в стекло (рис. 3.36, а), то угол преломления всегда будет меньшим, чем угол падения (Преломление света в физике - формулы и определения с примерами). Если же луч света направить из стекла в воздух (рис. 3.36, б),

то угол преломления всегда будет большим, чем  угол падения (Преломление света в физике - формулы и определения с примерами).

Напомним, что оптическая плотность стекла больше оптической плотности воздуха, и сформулируем закономерности преломления света.

  1. Луч падающий, луч преломленный и перпендикуляр к границе раздела двух сред, восставленный в точке падения луча, лежат в одной плоскости.
  2. Существуют такие соотношения между углом падения и углом преломления
  • а)    в случае увеличения угла падения увеличивается и угол преломления
  • б)    если луч света переходит из среды с меньшей оптической плотностью в среду с большей оптической плотностью, то угол преломления будет меньше, чем угол падения
  • в)    если луч света переходит из среды с большей оптической плотностью в среду с меньшей оптической плотностью, то угол преломления будет большим, чем угол падения.

(Следует отметить, что в старших классах, после изучения курса тригонометрии, вы глубже познакомитесь с преломлением света и узнаете о нем на уровне законов.)
 

Объясняем преломлением света некоторые оптические явления

Когда мы, стоя на берегу водоема, стараемся на глаз определить его глубину, она всегда кажется меньшей, чем есть на самом деле. Это явление объясняется преломлением света (рис. 3.37).

Следствием преломления света в атмосфере Земли является тот факт, что мы видим Солнце и звезды немного выше их реального положения (рис. 3.38). Преломлением света можно объяснить еще много природных явлений: возникновение миражей, радуги и др.

Явление преломления света является основой работы многочисленных оптических устройств (рис. 3.39). С некоторыми из них мы познакомимся в следующих параграфах, с некоторыми — в ходе дальнейшего изучения физики.
Преломление света в физике - формулы и определения с примерами

Преломление света в физике - формулы и определения с примерами

Итоги:

Световой пучок, падая на границу раздела двух сред, имеющих разную оптическую плотность, делится на два пучка. Один из них — отраженный — отражается от поверхности, подчиняясь законам отражения света. Второй — преломленный — проходит через границу раздела в другую среду, изменяя свое направление.

Причина преломления света — изменение скорости света в случае перехода из одной среды в другую. Если во время перехода света из одной среды в другую скорость света уменьшилась, то говорят, что свет перешел из среды с меньшей оптической плотностью в среду с большей оптической плотностью, и наоборот.

Преломление света происходит по определенным законам.

Преломление света в физике - формулы и определения с примерами

Преломление света

Преломление света в физике - формулы и определения с примерамиПреломление света в физике - формулы и определения с примерами
Почему ноги человека, зашедшего в воду, кажутся короче (рис. 250)? Дно бассейна мы видим ближе к поверхности, чем есть в действительности. Ложка в стакане на уровне поверхности воды (рис. 251) кажется переломленной. Как объяснить эти явления?

Когда пучок света падает на границу раздела двух прозрачных сред, часть его отражается, а часть переходит в другую среду, изменяя свое направление (рис. 252).

Преломление света в физике - формулы и определения с примерами

Изменение направления распространения света при переходе его через границу раздела двух сред называется преломлением.

Каким законам подчиняется преломление света?

Рассмотрим опыт. В центре оптического диска закрепим стеклянный полудиск (рис. 253), направим на него узкий пучок света (луч 1). Луч 3 — преломленный луч.

Угол Преломление света в физике - формулы и определения с примерами между перпендикуляром, проведенным в точку падения к границе раздела двух сред, и преломленным лучом называется углом преломления.

Сравнив углы Преломление света в физике - формулы и определения с примерами (см. рис. 253), мы видим, что угол преломления Преломление света в физике - формулы и определения с примерами меньше угла падения Преломление света в физике - формулы и определения с примерами

Увеличим угол падения (рис. 254). Угол преломления тоже увеличивается, но он по-прежнему меньше угла падения.

Преломление света в физике - формулы и определения с примерами

Если стекло заменить водой и пустить световой луч и под тем же углом Преломление света в физике - формулы и определения с примерами (рис. 255, а), что и на стеклянный полудиск, то угол преломления Преломление света в физике - формулы и определения с примерами в воде будет несколько больше, чем в стекле, но меньше угла падения: Преломление света в физике - формулы и определения с примерами Сравним скорости света в воздухе, воде и стекле: Преломление света в физике - формулы и определения с примерами т. е. стекло оптически более плотная среда, чем вода, а вода — чем воздух. Следовательно, при переходе луча из оптически менее плотной в оптически более плотную среду угол преломления меньше угла падения.

А если луч переходит из воды в воздух?

Из опыта (рис. 255, б) видно, что угол Преломление света в физике - формулы и определения с примерами больше угла Преломление света в физике - формулы и определения с примерами Значит, если свет переходит из среды оптически более плотной в оптически менее плотную, то угол преломления больше угла падения. Этот вывод логически следует из свойства обратимости, которое характерно не только для падающего и отраженного лучей, но и для падающего и преломленного лучей.

Из результатов проведенных опытов следует.

  1. Луч падающий и луч преломленный лежат в одной плоскости с перпендикуляром, проведенным в точку падения луча к границе раздела двух сред.
  2. Угол преломления меньше утла падения при переходе луча из оптически менее плотной среды в оптически более плотную среду. Угол преломления больше угла падения, если луч переходит из оптически более плотной среды в оптически менее плотную.

Эти два главных положения выражают суть явления преломления света. Однако, когда луч надает перпендикулярно на границу раздела двух сред Преломление света в физике - формулы и определения с примерамион не испытывает преломления, что можно подтвердить опытом (рис. 256).
Преломление света в физике - формулы и определения с примерами

Главные выводы:

  1. При переходе из одной среды в другую световой луч на границе раздела сред в большинстве случаев испытывает преломление (изменяет направление).
  2. Луч, падающий перпендикулярно к границе раздела двух сред, не испытывает преломления.
  3. Если луч переходит из оптически менее плотной среды в оптически более плотную, угол преломления меньше угла падения Преломление света в физике - формулы и определения с примерами При переходе луча из оптически более плотной среды в менее плотную угол преломления больше угла падения Преломление света в физике - формулы и определения с примерами

Преломление света на границе разделения двух сред. Закон преломления света

Еще в древние времена люди утверждали, что палка, опущенная в воду, на границе воздух-вода будто сломана. Вынув из воды, она оказывается целой. Так человек впервые столкнулся с явлением преломления света.

Первым это явление начал изучать древнегреческий естествоиспытатель Клеомед (I в. н. э.). Он установил, что луч света, распространяющийся под углом с менее плотной оптической среды в более плотную, например из воздуха в воду, изменяет свое направление, то есть преломляется. Клеомед говорил, что под определенным углом мы не будем видеть предмет, лежащий на дне сосуда (рис. 135), но если налить в сосуд воды, предмет будет видно.

Преломление света в физике - формулы и определения с примерами

Таким образом, по мнению Клеомеда, благодаря преломлению лучей можно видеть Солнце, зашедшее за горизонт.

Другой древнегреческий ученый Клавдий Птоломей (II в. н. э.) опытным путем определил величину, характеризующую преломление лучей света при переходе их из воздуха в воду, из воздуха в стекло и из воды в стекло.

Преломление света в физике - формулы и определения с примерами

Опыт 1. Направим луч света на тонкостенный сосуд с подкрашенной водой, который имеет форму прямоугольного параллелепипеда. Мы видим, что на границе двух сред луч света изменяет свое направление: отражается и преломляется (рис. 136, а).

Изменение направления распространения света при его переходе через границы разделения двух оптически прозрачных сред называют преломлением света.

Выполним чертеж (рис. 136, б). Опыт показывает, что угол отражения света Преломление света в физике - формулы и определения с примерами равен углу падения света а, а при переходе луча из воздуха в воду угол преломления света Преломление света в физике - формулы и определения с примерами(гамма) меньше угла падения света а. Кроме того, видим, что падающий и преломленный лучи света лежат в одной плоскости с перпендикуляром, проведенным к поверхности разделения двух сред в точку падения света. При переходе луча света из воды в воздух угол преломления света Преломление света в физике - формулы и определения с примерамибольше угла падения света Преломление света в физике - формулы и определения с примерами.

Этот опыт показывает, что при переходе светового луча с одной среды в другую: падающий и преломленный лучи света лежат в одной плоскости с перпендикуляром, проведенным к плоскости разделения двух сред в точку падения луча света; в зависимости от того, с какой среды в какую переходит луч света, угол преломления луча света может быть больше или меньше угла падения света.

Разные среды по-разному преломляют световые лучи. Например, алмаз преломляет лучи света больше, чем вода или стекло.

Среда, преломляющая свет, должна быть прозрачной, то есть такой, чтобы сквозь нее проходили лучи света.

Световые лучи преломляются, поскольку они распространяются в разных средах (телах) с неодинаковой скоростью. В воздухе скорость распространения света больше, чем в воде, в воде больше, чем в стекле.

Опыт 2. Поместим в сосуд с водой специальный источник света, от которого в разные стороны распространяются лучи света (рис. 137). Луч света, падающий перпендикулярно к границе вода-воздух, не преломляется.

Преломление света в физике - формулы и определения с примерами

Лучи света, падающие под разными углами к поверхности воды, преломляются по-разному. Но есть лучи света, которые вообще не переходят из воды в воздух, а полностью отражаются от ее поверхности. Явление, когда лучи света не выходят из среды и полностью отражаются внутрь, называют полным внутренним отражением света.

Явление полного внутреннего отражения света используют в специальных приборах — световодах. Световоды (рис. 138) широко применяют для передачи изображений предметов с любого места на любые расстояния.
Преломление света в физике - формулы и определения с примерами

Пример №1

1.    Какой из углов больше — угол падения или угол преломления, если свет переходит: а) из воды в воздух; б) из воздуха в стекло; в) из воды в стекло?

Ответ: а) угол падения; б) угол падения; в) угол преломления.

Пример №2

2.    В стакан с водой вставили трубку для сока. Как объяснить явление, изображенное на рисунке 145?

Преломление света в физике - формулы и определения с примерами

Ответ: если смотреть на рисунок, то видим, что трубка для сока кажется сломанной. Это объясняется законами преломления света.

Закон преломления света и показатель преломления

Геометрической оптикой называют раздел оптики, в которой изучаются законы распространения света в прозрачных средах на основе представления о нем как о совокупности световых лучей.

Под лучом понимают линию, вдоль которой переносится энергия электромагнитной волны. Условимся изображать оптические лучи графически с помощью геометрических лучей со стрелками. В геометрической оптике волновая природа света не учитывается.

Уже в начальные периоды оптических исследований были экспериментально установлены четыре основных закона геометрической оптики:

  • закон прямолинейного распространения света;
  • закон независимости световых лучей;
  • закон отражения световых лучей;
  • закон преломления световых лучей.

В этих законах использовались понятия световой пучок и световой луч, т. е. предполагалось, что пучок и луч бесконечно тонкие.

Световые пучки получают при пропускании светового излучения, идущего от удаленного источника, через отверстие (диафрагму) в экране I (рис. 52). Эксперименты показывают, что если диаметр D гораздо больше длины световой волны Преломление света в физике - формулы и определения с примерами и расстояние l от отверстия до экрана велико по сравнению с размером диафрагмы (l Преломление света в физике - формулы и определения с примерами D), то выходящий из диафрагмы пучок является параллельным. Для него на не слишком больших расстояниях l от экрана выполняется неравенство Преломление света в физике - формулы и определения с примерами

Преломление света в физике - формулы и определения с примерами

Если же диаметр диафрагмы или размеры предмета оказываются сравнимы с длиной световой волны, то выходящий световой пучок становится расходящимся, свет проникает в область геометрической тени, происходит дифракция света, т. е. проявляется волновой характер светового излучения. Следует отметить, что дифракция будет наблюдаться на очень больших расстояниях от экрана (Преломление света в физике - формулы и определения с примерами) даже при диаметре светового отверстия Преломление света в физике - формулы и определения с примерами.

Таким образом, луч — это направление, перпендикулярное фронту волны, в котором она переносит энергию.

Лучи, выходящие из одной точки, называют расходящимися, а собирающиеся в одной точке, — сходящимися. Примером расходящихся лучей может служить наблюдаемый свет далеких звезд, а примером сходящихся — совокупность лучей, попадающих в зрачок нашего глаза от различных предметов.

Для изучения свойств световых волн необходимо знать как закономерности их распространения в однородной среде, так и закономерности отражения и преломления на границе раздела двух сред.

Преломление света в физике - формулы и определения с примерами

Рассмотрим процессы, происходящие при падении плоской световой волны на плоскую поверхность раздела однородных изотропных и прозрачных сред при условии, что размеры поверхности раздела намного больше длины волны падающего излучения.

Пусть на плоскую поверхность раздела LM двух сред падает плоская световая волна, фронт которой АВ (рис. 53). Если угол падения а отличен от нуля, то различные точки фронта АВ волны достигнут границы раздела LM не одновременно.

Согласно принципу Гюйгенса точка Преломление света в физике - формулы и определения с примерами которой фронт волны достигнет раньше всего (см. рис. 53), станет источником вторичных волн. Вторичные волны будут распространяться со скоростью v и за промежуток времени Преломление света в физике - формулы и определения с примерами за который точка фронта Преломление света в физике - формулы и определения с примерами, достигнет границы раздела двух сред (точки Преломление света в физике - формулы и определения с примерами), вторичные волны из точки Преломление света в физике - формулы и определения с примерами пройдут расстояние Преломление света в физике - формулы и определения с примерами Падающая и возникающие вторичные волны распространяются в одной и той же среде, поэтому их скорости одинаковы, и они пройдут одинаковые расстояния Преломление света в физике - формулы и определения с примерами

Касательная, проведенная из точки Преломление света в физике - формулы и определения с примерами к полуокружности радиусом Преломление света в физике - формулы и определения с примерамиявляется огибающей вторичных волн и дает положение фронта волны через промежуток времени Преломление света в физике - формулы и определения с примерами. Затем он перемещается в направлении Преломление света в физике - формулы и определения с примерами.

Из построения следует, что Преломление света в физике - формулы и определения с примерами С учетом определений угла падения Преломление света в физике - формулы и определения с примерамии угла отражения Преломление света в физике - формулы и определения с примерами находим, что Преломление света в физике - формулы и определения с примерами как углы с взаимно перпендикулярными сторонами. Следовательно, угол отражения равен углу падения (Преломление света в физике - формулы и определения с примерами = Преломление света в физике - формулы и определения с примерами). Таким образом, исходя из волновой теории света на основании принципа Гюйгенса получен закон отражения света.

Преломление света в физике - формулы и определения с примерами

Рассмотрим, что будет происходить во второй среде (рис. 54), считая, что скорость Преломление света в физике - формулы и определения с примерами распространения света в ней меньше, чем в первой (Преломление света в физике - формулы и определения с примерами<Преломление света в физике - формулы и определения с примерами)-Фронт падающей волны АВ будет перемещаться со скоростью у, по направлению Преломление света в физике - формулы и определения с примерами. К моменту времени Преломление света в физике - формулы и определения с примерами когда точка В фронта достигнет границы раздела двух сред (точка Преломление света в физике - формулы и определения с примерами), вторичная волна из точки Преломление света в физике - формулы и определения с примерами (согласно принципу Гюйгенса) пройдет расстояние Преломление света в физике - формулы и определения с примерами Фронт волны, распространяющейся во второй среде, можно получить, проводя прямую линию, касательную к полуокружности с центром в точке Преломление света в физике - формулы и определения с примерами.

Из построения видно, что Преломление света в физике - формулы и определения с примерами как углы с взаимно перпендикулярными сторонами.

Из треугольника Преломление света в физике - формулы и определения с примерами находим Преломление света в физике - формулы и определения с примерами и из треугольника Преломление света в физике - формулы и определения с примерами — Преломление света в физике - формулы и определения с примерамиоткуда получаем соотношение

Преломление света в физике - формулы и определения с примерами

Из него следует закон преломления

Преломление света в физике - формулы и определения с примерами

Вспомним, что абсолютным показателем преломления называется отношение скорости распространения световой волны в вакууме с к ее скорости распространения в данной среде v:

Преломление света в физике - формулы и определения с примерами

С учетом этого соотношения закон преломления принимает вид:

Преломление света в физике - формулы и определения с примерами

Таким образом, исходя из волновой теории света, получен закон преломления световых волн:

Можно записать закон преломления и в другом виде:

Преломление света в физике - формулы и определения с примерами

Для наблюдения явления преломления света достаточно поместить карандаш в стакан с водой и посмотреть на него со стороны — карандаш будет казаться «надломленным» (преломленным) (рис. 55).

Преломление света в физике - формулы и определения с примерами

Первые упоминания о преломлении света в воде и стекле встречаются в труде Клавдия Птолемея «Оптика», вышедшем в свет во II в. н. э. Закон преломления света был экспериментально установлен в 1620 г. голландским ученым Виллебродом Снеллиусом. Независимо от Снеллиуса закон преломления был также открыт Рене Декартом.

Отметим, что причиной преломления волн, т. е. изменения направления распространения волн на границе раздела двух сред, является изменение скорости распространения электромагнитных волн при переходе излучения из одной среды в другую.

Как следует из закона преломления, при переходе света из оптически более плотной среды I (с большим абсолютным показателем преломления Преломление света в физике - формулы и определения с примерами) (рис. 56) в оптически менее плотную среду II (с меньшим показателем преломления Преломление света в физике - формулы и определения с примерами) угол преломления у становится больше угла падения Преломление света в физике - формулы и определения с примерами.

Преломление света в физике - формулы и определения с примерами

По мере увеличения угла падения при некотором его значении Преломление света в физике - формулы и определения с примерами угол преломления станет Преломление света в физике - формулы и определения с примерами = 90°, т. е. свет не будет попадать во вторую среду.

Энергия преломленной волны при этом станет равной нулю, а энергия отраженной волны будет равна энергии падающей. Следовательно, начиная с этого угла падения, вся световая энергия отражается от границы раздела этих сред в среду I.

Это явление называется полным отражением (см. рис. 56). Угол Преломление света в физике - формулы и определения с примерами, при котором начинается полное отражение, называется предельным углом полного отражения. Он определяется из закона преломления при условии, что угол преломления Преломление света в физике - формулы и определения с примерами = 90°:

Преломление света в физике - формулы и определения с примерами

Таким образом, при углах падения больших Преломление света в физике - формулы и определения с примерами преломленная волна отсутствует.

В 1954 г. белорусским физиком, академиком Федором Ивановичем Федоровым было теоретически предсказано новое физическое явление — боковое смещение светового пучка при полном отражении. В 1969 г. французским физиком К- Эмбером оно было подтверждено экспериментально и получило название «сдвиг Федорова». Федоровым был развит новый бескоординатный метод описания оптических свойств кристаллов. На его основе разработана общая теория оптических свойств поглощающих кристаллов.

Полное отражение

Изменение направления распространения луча света при прохождении через границу раздела двух сред называется преломлением света.

Геометрической оптикой называют раздел оптики, в котором изучаются законы распространения оптического излучения на основе представления о световых лучах.

Под лучом понимают линию, вдоль которой переносится энергия электромагнитной волны. Условимся изображать световые лучи графически с помощью геометрических линий со стрелками. В геометрической оптике волновая природа света не учитывается.

Геометрическому лучу на практике соответствует тонкий световой пучок, получаемый при пропускании светового излучения, идущего от удаленного источника, через отверстие (диафрагму) в экране (рис. 69).

Преломление света в физике - формулы и определения с примерами

Таким образом, следует различать геометрический луч (математическое понятие) и световой пучок (материальный объект), получаемый от источника света.

Уже в начальные периоды оптических исследований были экспериментально установлены четыре основных закона геометрической оптики:

  • закон прямолинейного распространения света;
  • закон независимости световых лучей;
  • закон отражения световых лучей;
  • закон преломления световых лучей.

Световой поток можно разделить на отдельные световые пучки, выделяя их при помощи диафрагм. Действие выделенных световых пучков оказывается независимым друг от друга, т. е. эффект, производимый отдельным пучком, не зависит от того, действуют одновременно с ним другие пучки или нет.

Световые пучки получают при пропускании светового излучения, идущего от удаленного источника, через отверстие (диафрагму) в экране Преломление света в физике - формулы и определения с примерами (рис. 70). Для того чтобы можно было пренебречь дифракционным расширением пучка, должно выполняться условие:

Преломление света в физике - формулы и определения с примерами

Преломление света в физике - формулы и определения с примерами

где Преломление света в физике - формулы и определения с примерами — размер препятствия или отверстия, на котором свет дифрагирует, Преломление света в физике - формулы и определения с примерами — длина световой волны, Преломление света в физике - формулы и определения с примерами — расстояние от препятствия до места наблюдения дифракционной картины.

В этом случае выходящий из диафрагмы пучок будет оставаться неизменным, и он называется параллельным.

Соотношение (1) выполняется, когда длина световой волны стремиться к нулю Преломление света в физике - формулы и определения с примерами Поэтому геометрическая оптика является предельным приближенным случаем волновой оптики.

gtftf Если диаметр диафрагмы или размеры предмета оказываются сравнимы  с длиной световой волны Преломление света в физике - формулы и определения с примерами то выходящий световой пучок становится расходящимся, свет проникает в область геометрической тени, происходит дифракция света, т. е. проявляется волновой характер светового излучения.

Лучи, выходящие из одной точки, называют расходящимися, а собирающиеся в одной точке — сходящимися. Примером расходящихся лучей может служить наблюдаемый свет далеких звезд, а примером сходящихся — совокупность лучей, попадающих в зрачок нашего глаза от различных предметов.

Для изучения свойств световых волн необходимо знать закономерности их распространения в однородной среде, а также закономерности отражения и преломления на границе раздела двух сред.

Рассмотрим падение плоской световой волны на плоскую поверхность раздела однородных изотропных и прозрачных сред при условии, что размеры поверхности раздела намного больше длины волны падающего излучения.

Пусть на плоскую поверхность раздела Преломление света в физике - формулы и определения с примерами двух сред падает плоская световая волна, фронт которой Преломление света в физике - формулы и определения с примерами (рис. 71). Если угол падения Преломление света в физике - формулы и определения с примерами отличен от нуля, то различные точки фронта Преломление света в физике - формулы и определения с примерами волны достигнут границы раздела Преломление света в физике - формулы и определения с примерами не одновременно.

Рассмотрим, что будет происходить во второй среде, считая, что модуль скорости Преломление света в физике - формулы и определения с примерами распространения света в ней меньше, чем в первой Преломление света в физике - формулы и определения с примерами (см. рис. 71).

Фронт падающей волны Преломление света в физике - формулы и определения с примерами будет перемещаться со скоростью, модуль которой Преломление света в физике - формулы и определения с примерами по направлению Преломление света в физике - формулы и определения с примерами К моменту времени (за промежуток времени Преломление света в физике - формулы и определения с примерами

когда точка Преломление света в физике - формулы и определения с примерами фронта достигнет границы раздела двух сред (точка Преломление света в физике - формулы и определения с примерами вторичная волна из точки Преломление света в физике - формулы и определения с примерами (согласно принципу Гюйгенса) пройдет расстояние Преломление света в физике - формулы и определения с примерами Фронт волны, распространяющейся во второй среде, можно получить, проводя прямую линию, касательную к полуокружности с центром в точке Преломление света в физике - формулы и определения с примерами

Из построения видно, что Преломление света в физике - формулы и определения с примерами как углы с взаимно перпендикулярными сторонами. Из Преломление света в физике - формулы и определения с примерами находим: Преломление света в физике - формулы и определения с примерамиПреломление света в физике - формулы и определения с примерами
Отсюда:
Преломление света в физике - формулы и определения с примерами
Из этого выражения следует закон преломления:

Преломление света в физике - формулы и определения с примерами

Напомним, что абсолютным показателем преломления называется отношение модуля скорости распространения световой волны в вакууме с к модулю скорости распространения в данной среде Преломление света в физике - формулы и определения с примерами

С учетом этого соотношения закон преломления принимает вид:

Преломление света в физике - формулы и определения с примерами

Величина

Преломление света в физике - формулы и определения с примерами

равная отношению абсолютных показателей преломления Преломление света в физике - формулы и определения с примерами второй и Преломление света в физике - формулы и определения с примерами первой сред, называется относительным показателем преломления второй среды относительно первой. В отличие от абсолютного показателя преломления относительный показатель преломления может быть и меньше единицы, если Преломление света в физике - формулы и определения с примерами

Таким образом, исходя из волновой теории света, получен закон преломления световых волн:

отношение синуса угла падения к синусу угла преломления есть величина постоянная для двух данных сред и равная относительному показателю преломления второй среды относительно первой;

лучи, падающий и преломленный, лежат в одной плоскости с перпендикуляром, проведенным в точке падения луча к плоскости границы раздела двух сред.

Перепишем закон преломления в следующем виде:

Преломление света в физике - формулы и определения с примерами

При такой записи закона преломления не надо запоминать абсолютный показатель преломления какой среды стоит в числителе, а какой — в знаменателе.

Необходимо всегда умножать абсолютный показатель преломления на синус угла, относящийся к одной и той же среде.

Для наблюдения явления преломления света достаточно поместить карандаш в стакан с водой и посмотреть на него со стороны — карандаш будет казаться «надломленным» (преломленным) (рис. 72), оставаясь при этом совершенно целым.

Преломление света в физике - формулы и определения с примерами

Первые упоминания о преломлении света в воде и стекле встречаются в труде Клавдия Птолемея «Оптика», вышедшем в свет во II в. н. э.

Закон преломления света был экспериментально установлен в 1620 г. голландским ученым Виллебродом Снеллиусом. Заметим, что независимо от Снеллиуса закон преломления был также открыт Рене Декартом.

Причиной преломления волн, т. е. изменения направления распространения волн на границе раздела двух сред, является 
Преломление света в физике - формулы и определения с примерами

изменение модуля скорости распространения электромагнитных волн при переходе излучения из одной среды в другую.

Как следует из закона преломления, при переходе света из оптически более плотной среды 1 (с большим абсолютным показателем преломления Преломление света в физике - формулы и определения с примерами в оптически менее плотную среду II (с меньшим показателем преломления Преломление света в физике - формулы и определения с примерами угол преломления у становится больше угла падения Преломление света в физике - формулы и определения с примерами (рис. 73).

По мере увеличения угла падения, при некотором его значении Преломление света в физике - формулы и определения с примерами угол преломления станет Преломление света в физике - формулы и определения с примерами т. е. свет не будет попадать во вторую среду. Энергия преломленной волны при этом станет равной нулю, а энергия отраженного излучения будет равна энергии падающего. Следовательно, начиная с этого угла падения, вся световая энергия полностью отражается от границы раздела этих сред в среду Преломление света в физике - формулы и определения с примерами

Это явление называется полным отражением света (см. рис. 73). Угол Преломление света в физике - формулы и определения с примерами при котором возникает полное отражение, называется предельным углом полного отражения. Он определяется из закона преломления при условии, что угол преломления: Преломление света в физике - формулы и определения с примерами

Преломление света в физике - формулы и определения с примерами

Таким образом, преломленная волна отсутствует при углах падения, больших предельного угла Преломление света в физике - формулы и определения с примерами Например, для границы вода Преломление света в физике - формулы и определения с примерами — воздух предельный угол полного отражения Преломление света в физике - формулы и определения с примерами для границы алмаз Преломление света в физике - формулы и определения с примерами — воздух — Преломление света в физике - формулы и определения с примерами

Явление полного отражения используют в волоконной оптике для передачи света и изображения по пучкам прозрачных гибких световодов (рис. 74), а также в отражательных призмах различных оптических приборов. В волоконно-оптических устройствах, в которых свет распространяется по тонким световодам, стеклянная световедущая жила покрыта слоем вещества с меньшим показателем преломления.
Преломление света в физике - формулы и определения с примерами

В 2009 г. китайский ученый Чарльз Пао удостоен Нобелевской премии за выдающийся вклад в исследование световодов для оптической связи. В 1954 г. белорусским физиком, академиком Федором Ивановичем Федоровым было теоретически предсказано новое физическое явление — поперечное смещение (перпендикулярно плоскости падения) светового пучка при его полном отражении. Это смещение меньше длины волны, и для его наблюдения световой пучок должен быть ограниченным в поперечном направлении. В 1969 г. французским физиком К. Эмбером оно было подтверждено экспериментально и получило название «сдвиг Федорова».

  • Заказать решение задач по физике

Пример №3

Определите угол падения Преломление света в физике - формулы и определения с примерами луча на стеклянную пластинку с показателем преломления Преломление света в физике - формулы и определения с примерами если между отраженным и преломленным лучами угол Преломление света в физике - формулы и определения с примерами

Дано: 

Преломление света в физике - формулы и определения с примерами

Преломление света в физике - формулы и определения с примерами

Решение

Из закона преломления находим:

Преломление света в физике - формулы и определения с примерами

Из геометрического построения (рис. 75) следует, что углы отражения и преломления связаны соотношением: 

Преломление света в физике - формулы и определения с примерами

Преломление света в физике - формулы и определения с примерами

Отсюда: 

Преломление света в физике - формулы и определения с примерами

Подставляем найденный угол Преломление света в физике - формулы и определения с примерами в формулу закона преломления и с учетом закона отражения Преломление света в физике - формулы и определения с примерами определяем искомый угол падения:
Преломление света в физике - формулы и определения с примерами

 Отсюда

Преломление света в физике - формулы и определения с примерами

Ответ: Преломление света в физике - формулы и определения с примерами

Преломление света на границе раздела двух сред

В одном из древнегреческих трактатов описан опыт: «Надо встать так, чтобы плоское кольцо, расположенное на дне сосуда, спряталось за его краем. Затем, не меняя положения глаз, налить в сосуд воду. Свет преломится на поверхности воды, и кольцо станет видимым». Такой «фокус» вы можете показать своим друзьям и сейчас (см. рис. 12.1), а вот объяснить его сможете только после изучения данного параграфа.

Преломление света в физике - формулы и определения с примерами

Рис. 12.1. «Фокус» с монетой. Если в чашке нет воды, мы не видим монету, лежащую на ее дне (а); если же налить воду, дно чашки будто поднимется и монета станет видимой (б)

Законы преломления света

Направим узкий пучок света на плоскую поверхность прозрачного стеклянного полуцилиндра, закрепленного на оптической шайбе. Свет не только отразится от поверхности полуцилиндра, но и частично пройдет сквозь стекло. Это означает, что при переходе из воздуха в стекло направление распространения света изменяется (рис. 12.2).

Преломление света в физике - формулы и определения с примерами

Рис. 12.2. Наблюдение преломления света при его переходе из воздуха в стекло: Преломление света в физике - формулы и определения с примерами — угол падения; Преломление света в физике - формулы и определения с примерами — угол отражения; Преломление света в физике - формулы и определения с примерами — угол преломления

Изменение направления распространения света на границе раздела двух сред называют преломлением света.

Угол Преломление света в физике - формулы и определения с примерами (гамма), который образован преломленным лучом и перпендикуляром к границе раздела двух сред, проведенным через точку падения луча, называют углом преломления.

Проведя ряд опытов с оптической шайбой, заметим, что с увеличением угла падения угол преломления тоже увеличивается, а с уменьшением угла падения угол преломления уменьшается (рис. 12.3). Если же свет падает перпендикулярно границе раздела двух сред (угол падения Преломление света в физике - формулы и определения с примерами направление распространения света не изменяется.

Преломление света в физике - формулы и определения с примерами

Рис. 12.3. Установление законов преломления света: при уменьшении угла падения Преломление света в физике - формулы и определения с примерамиугол преломления тоже уменьшается Преломление света в физике - формулы и определения с примерами при этом Преломление света в физике - формулы и определения с примерами

Первое упоминание о преломлении света можно найти в трудах древнегреческого философа Аристотеля (IV в. до н. э.), который задавался вопросом: «Почему палка в воде кажется сломанной?» А вот закон, количественно описывающий преломление света, был установлен только в 1621 г. голландским ученым Виллебрордом Снеллиусом (1580-1626).

Законы преломления света:

  1. Луч падающий, луч преломленный и перпендикуляр к границе раздела двух сред, проведенный через точку падения луча, лежат в одной плоскости.
  2. Отношение синуса угла падения к синусу угла преломления для двух данных сред является неизменной величиной:

Преломление света в физике - формулы и определения с примерами

где Преломление света в физике - формулы и определения с примерами — физическая величина, которую называют относительным показателем преломления среды 2 (среды, в которой свет распространяется после преломления) относительно среды 1 (среды, из которой свет падает).

Причина преломления света

Так почему свет, переходя из одной среды в другую, изменяет свое направление?

Дело в том, что в разных средах свет распространяется с разной скоростью, но всегда медленнее, чем в вакууме. Например, в воде скорость света в 1,33 раза меньше, чем в вакууме; когда свет переходит из воды в стекло, его скорость уменьшается еще в 1,3 раза; в воздухе скорость распространения света в 1,7 раза больше, чем в стекле, и лишь немного меньше (примерно в 1,0003 раза), чем в вакууме.

Именно изменение скорости распространения света при переходе из одной прозрачной среды в другую является причиной преломления света.

Принято говорить об оптической плотности среды: чем меньше скорость распространения света в среде (чем больше показатель преломления), тем больше оптическая плотность среды.

Физический смысл показателя преломления

Относительный показатель преломления Преломление света в физике - формулы и определения с примерами показывает, во сколько раз скорость распространения света в среде 1 больше (или меньше) скорости распространения света в среде 2:

Преломление света в физике - формулы и определения с примерами

Вспомнив второй закон преломления света: Преломление света в физике - формулы и определения с примерами имеем:

Преломление света в физике - формулы и определения с примерами

Проанализировав последнюю формулу, делаем выводы:

  1. чем больше на границе раздела двух сред изменяется скорость распространения света, тем больше свет преломляется;
  2. если луч света переходит в среду с большей оптической плотностью (то есть скорость света уменьшается: Преломление света в физике - формулы и определения с примерами то угол преломления меньше угла падения: Преломление света в физике - формулы и определения с примерами (см., например, рис. 12.2, 12.3);
  3. если луч света переходит в среду с меньшей оптической плотностью (то есть скорость света увеличивается: Преломление света в физике - формулы и определения с примерами то угол преломления больше угла падения: Преломление света в физике - формулы и определения с примерами (рис. 12.4).

Обычно скорость распространения света в среде сравнивают со скоростью его распространения в вакууме. Когда свет попадает в среду из вакуума, показатель преломления Преломление света в физике - формулы и определения с примерами называют абсолютным показателем преломления.

Абсолютный показатель преломления показывает, во сколько раз скорость

распространения света в среде меньше, чем в вакууме:

Преломление света в физике - формулы и определения с примерами

где Преломление света в физике - формулы и определения с примерами — скорость распространения света в вакууме Преломление света в физике - формулы и определения с примерами — скорость распространения света в среде.

Преломление света в физике - формулы и определения с примерами

Рис. 12.4. При переходе света из среды с большей оптической плотностью в среду с меньшей оптической плотностью угол преломления больше угла падения Преломление света в физике - формулы и определения с примерами

Среда

Абсолютный показатель

преломления Преломление света в физике - формулы и определения с примерами

Воздух 1,0003
Лед 1,31
Вода 1,33
Бензин 1,50
Стекло 1,43-2,17
Кварц 1,54
Алмаз 2,42

Скорость распространения света в вакууме больше, чем в любой среде, поэтому абсолютный показатель преломления всегда больше единицы (см. таблицу).

Обратите внимание: Преломление света в физике - формулы и определения с примерами поэтому, рассматривая переход света из воздуха в среду, будем считать, что относительный показатель преломления среды равен абсолютному.

Явление преломления света используется в работе многих оптических устройств. О некоторых из них вы узнаете позже.

Явление полного внутреннего отражения света

Рассмотрим случай, когда свет переходит из среды с большей оптической плотностью в среду с меньшей оптической плотностью (рис. 12.5). Видим, что при увеличении угла падения Преломление света в физике - формулы и определения с примерами угол преломления Преломление света в физике - формулы и определения с примерами приближается к 90°, яркость преломленного пучка уменьшается, а яркость отраженного, наоборот, увеличивается. Понятно, что если и дальше увеличивать угол падения, то угол преломления достигнет 90°, преломленный пучок исчезнет, а падающий пучок полностью (без потерь энергии) вернется в первую среду — свет полностью отразится.

Преломление света в физике - формулы и определения с примерами

Рис. 12.5. Если свет попадает из стекла в воздух, то при увеличении угла падения угол преломления приближается к 90°, а яркость преломленного пучка уменьшается

Явление, при котором преломление света отсутствует (свет полностью отражается от среды с меньшей оптической плотностью), называют полным внутренним отражением света.

Явление полного внутреннего отражения света хорошо знакомо тем, кто плавал под водой с открытыми глазами (рис. 12.6).

Преломление света в физике - формулы и определения с примерами

Рис. 12.6. Наблюдателю, находящемуся под водой, часть поверхности воды кажется блестящей, будто зеркало

Ювелиры много веков используют явление полного внутреннего отражения, чтобы повысить привлекательность драгоценных камней. Естественные камни огранивают — придают им форму многогранников: грани камня выполняют роль «внутренних зеркал», и камень «играет» в лучах падающего на него света.

Полное внутреннее отражение широко используют в оптической технике (рис. 12.7). Но главное применение этого явления связано с волоконной оптикой. Если в торец сплошной тонкой «стеклянной» трубки направить пучок света, после многократного отражения свет выйдет на ее противоположном конце независимо от того, какой будет трубка — изогнутой или прямой. Такую трубку называют световодом (рис. 12.8).

Преломление света в физике - формулы и определения с примерами

Рис. 12.7. Во многих оптических приборах направление распространения света изменяют с помощью призм полного отражения: а — призма поворачивает изображение; б — призма переворачивает изображение

Преломление света в физике - формулы и определения с примерами

Рис. 12.8. Распространение светового пучка в световоде

Световоды применяют в медицине для исследования внутренних органов (эндоскопия); в технике, в частности для выявления неисправностей внутри двигателей без их разборки; для освещения солнечным светом закрытых помещений и т. п. (рис. 12.9).

Преломление света в физике - формулы и определения с примерами

Рис. 12.9. Декоративный светильник со световодами

Но чаще всего световоды используют в качестве кабелей для передачи информации (рис. 12.10). «Стеклянный кабель» намного дешевле и легче медного, он практически не изменяет свои свойства под воздействием окружающей среды, позволяет передавать сигналы на большие расстояния без усиления. Сегодня волоконно-оптические линии связи стремительно вытесняют традиционные. Когда вы будете смотреть телевизор или пользоваться Интернетом, вспомните, что значительную часть своего пути сигнал проходит по «стеклянной дороге».

Преломление света в физике - формулы и определения с примерами

Рис. 12.10. Оптоволоконный кабель

Пример №4

Световой луч переходит из среды 1 в среду 2 (рис. 12.11, а). Скорость распространения света в среде 1 равна Преломление света в физике - формулы и определения с примерами Определите абсолютный показатель преломления среды 2 и скорость распространения света в среде 2.

Преломление света в физике - формулы и определения с примерами

Рис. 12.11. К задаче

Анализ физической проблемы

Из рис. 12.11, а видим, что на границе раздела двух сред свет преломляется, значит, скорость его распространения изменяется.

Выполним пояснительный рисунок (рис. 12.11, б), на котором:

1) изобразим лучи, приведенные в условии задачи; 2) проведем через точку падения луча перпендикуляр к границе раздела двух сред; 3) обозначим Преломление света в физике - формулы и определения с примерами угол падения и Преломление света в физике - формулы и определения с примерами— угол преломления.

Абсолютный показатель преломления — это показатель преломления относительно вакуума. Поэтому для решения задачи следует вспомнить значение скорости распространения света в вакууме и найти скорость распространения света в среде Преломление света в физике - формулы и определения с примерами

Чтобы найти Преломление света в физике - формулы и определения с примерами определим синус угла падения и синус угла преломления.

Дано:

Преломление света в физике - формулы и определения с примерами

Найти:

Преломление света в физике - формулы и определения с примерами

Поиск математической модели, решение

По определению абсолютного показателя преломления:

Преломление света в физике - формулы и определения с примерами

Поскольку Преломление света в физике - формулы и определения с примерами то Преломление света в физике - формулы и определения с примерами

Из рис. 12.11, б видим, что Преломление света в физике - формулы и определения с примерами где Преломление света в физике - формулы и определения с примерами радиус окружности. Найдем значения искомых величин:

Преломление света в физике - формулы и определения с примерами

Анализ решения. По условию задачи угол падения больше угла преломления, и это значит, что скорость света в среде 2 меньше скорости света в среде 1. Следовательно, полученные результаты реальны.

Ответ: Преломление света в физике - формулы и определения с примерами

Подводим итоги:

Световой пучок, падая на границу раздела двух сред, разделяется на два пучка. Один из них — отраженный — отражается от поверхности, подчиняясь законам отражения света. Второй — преломленный — проходит во вторую среду, изменяя свое направление.

Законы преломления света:

  1. Луч падающий, луч преломленный и перпендикуляр к границе раздела двух сред, проведенный через точку падения луча, лежат в одной плоскости.
  2. Для двух данных сред отношение синуса угла падения Преломление света в физике - формулы и определения с примерами к синусу угла преломления Преломление света в физике - формулы и определения с примерами является неизменной величиной: Преломление света в физике - формулы и определения с примерами

Причина преломления света — изменение скорости его распространения при переходе из одной среды в другую. Относительный показатель преломления Преломление света в физике - формулы и определения с примерами показывает, во сколько раз скорость распространения света в среде 1 больше (или меньше), чем скорость распространения света в среде 2: Преломление света в физике - формулы и определения с примерами

Когда свет попадает в среду из вакуума, показатель преломления Преломление света в физике - формулы и определения с примерами называют абсолютным показателем преломления: Преломление света в физике - формулы и определения с примерами

Если при переходе света из среды 1 в среду 2 скорость распространения света уменьшилась (то есть показатель преломления среды 2 больше показателя преломления среды 1: Преломление света в физике - формулы и определения с примерами то говорят, что свет перешел из среды с меньшей оптической плотностью в среду с большей оптической плотностью (и наоборот).

  • Полное отражение
  • Дисперсия света
  • Электромагнитная природа света
  • Интерференция света
  • Освещенность в физике
  • Закон прямолинейного распространения света
  • Законы отражения света
  • Зеркальное и рассеянное отражение света

Основные законы геометрической оптики были известны задолго до установления физической природы света. Большая часть из них выводятся из общего принципа, описывающего поведение волн. Впервые этот принцип выдвинул современник Ньютона Христиан Гюйгенс.

Принцип Гюйгенса

Каждая точка среды, до которой дошло возмущение, сама становится источником вторичных волн.

Чтобы, зная положение волновой поверхности в момент времени t, найти ее положение в следующий момент времени t + ∆t, нужно каждую точку волновой поверхности рассматривать как источник вторичных волн. Поверхность, касательная ко всем вторичным волнам, представляет собой волновую поверхность в следующий момент времени. Этот принцип подходит для описания волн любой природы (световых, механических, электромагнитных и пр.).

https://sites.google.com/site/adrosk386/_/rsrc/1367760980443/home/volnovaa-priroda-sveta/princip-gujgensa/capture-20130504-120637.png

Для механических волн принцип Гюйгенса имеет наглядное толкование: частицы среды, до которых доходят колебания, колеблясь, приводят в движение соседние частицы среды, с которыми они взаимодействуют.

Закон прямолинейного распространения света

В оптически однородной среде свет распространяется прямолинейно.

Опытным доказательством этого закона служат резкие тени, отбрасываемые непрозрачными телами при освещении светом источника небольших размеров («точечного источника»).

Другим доказательством может служить известный опыт по прохождению света далекого источника сквозь небольшое отверстие, в результате чего образуется узкий световой пучок. Этот опыт приводит к представлению о световом луче как о геометрической линии, вдоль которой распространяется свет.

Внимание!

Законы геометрической оптики выполняются приближенно при условии, что размеры препятствий на пути световых волн много больше длины волны. Так, закон прямолинейного распространения света нарушается и понятие светового луча утрачивает смысл, если свет проходит через очень малые отверстия.

Пример №1. Здание, освещенное солнечными лучами, отбрасывает тень длиной L = 36 м. Вертикальный шест высотой h = 2,5 м отбрасывает тень длиной l = 3 м. Найдите высоту H здания.

Так как шест и здание расположены вертикально, они параллельны. Так как на них светит один и тот же источник света, то угол падения лучей одинаков. Следовательно, треугольники, образованные стеной зданий, лучом солнца и землей, а также землей, лучом солнца и шестом, подобны. Отсюда можно сделать вывод, что отношение высоты здания к высоте шеста будет отношению длины тени здания к длине тени шеста:

Hh=Ll

H2,5=363=12

H=12·2,5=30 (м)

Закон отражения света

Рассмотрим отражение плоской волны (см. рис. ниже).

https://lh3.googleusercontent.com/proxy/gK8hs-HtpwFdBZfFk59D9W3LRcl5tX_6yCwp9Oduj_30HFjyqWDRnFeMeqnDYT7tUmT6GqbVWx1ivd_HexOQldsoOk0

Пусть:

  • MN — отражающая поверхность.
  • A1A и B1B — два параллельных луча падающей плоской волны.
  • AC — волновая поверхность плоской волны.
  • α и γ— угол падения и отражения лучей A1A и B1B.

Определение

Плоская волна — волна, волновые поверхности которой представляют собой плоскости.

Угол падения — угол между падающим лучом и перпендикуляром к отражающей поверхности.

Угол отражения — угол между перпендикуляром к отражающей поверхности и отраженным лучом.

Волновую поверхность отраженной волны можно получить, если провести огибающую вторичных волн, центры которых лежат на границе раздела сред. Различные участки волновой поверхности AC достигают отражающей границы неодновременно. Возбуждение колебаний в точке A начинается раньше, чем в точке B, на время Δt=CBv (v — скорость волны).

В момент, когда волна достигнет точки B, и в этой точке начнется возбуждение колебаний, вторичная волна в точке A уже будет представлять собой полусферу радиусом r = AD = v∆t = CB. Радиусы вторичных волн от источников, находящихся между точками A и B, меняются так, как показано на рисунке выше.

Огибающей вторичных волн является плоскость DB, касательная к сферическим поверхностям. Она является волновой поверхностью отраженной волны. Отраженные лучи AA2 и BB2 перпендикулярны волновой поверхности DB. Между ними образуется угол γ, являющийся углом отражения.

Так как AD = CB и треугольники ADB и ACB прямоугольные, то углы DBA и CAB равны. Но угол α= CAB, а γ= DBA как углы с перпендикулярными сторонами. Следовательно, α=γ.

Закон отражения света

Угол падения равен углу отражения. Падающий луч, луч отраженный и перпендикуляр, восстановленный в точке падения, лежат в одной плоскости.

Пример №2. Луч света падает на плоское зеркало. Угол падения α равен 20°. Чему равен угол между падающим и отражённым лучами?

Поскольку, согласно закон отражения света, угол падения равен углу отражения, то угол между падающим и отражённым лучами равен удвоенному углу α. Следовательно, он равен 40°.

Закон преломления света

На границе двух разнородных сред свет меняет направление распространения. Часть его энергии возвращается в первую среду, то есть, происходит отражение света. Если же вторая среда прозрачна, то часть света проходит через границу, разделяющую первому и вторую среду. При этом он меняет свое направление. Это явление называется преломлением света.

Преломление света на границе двух сред легко продемонстрировать с помощью стакана, воды и карандаша. Если опустить карандаш в пустой стакан, то он будет выглядеть таким же прямым, как и всегда (см. рисунок слева). Если же опустить карандаш в стакан, заполненный водой, мы увидим, что его часть под водой будто бы «преломилась».

Закон преломления света, который определяет взаимное расположение луча падающего, луча преломленного и перпендикуляра, восстановленного в точке падения, был открыт опытным путем в XVII веке. Но его можно доказать, основываясь на принципе Гюйгенса.

Известно, что скорость света достигает максимального значения только в вакууме. При распространении в среде скорость света снижается. Преломление света при переходе из одной среды в другую вызвано различием в скоростях распространения света в той и другой среде. Обозначим скорость распространения волны в первой среде как v1, а во второй — как v2.

Пусть на плоскую границу раздела двух сред (к примеру, из воздуха в воду) падает плоская световая волна (см. рисунок выше). Волновая поверхность AC перпендикулярна лучам A1A и B1B. Поверхности MN сначала достигнет луч A1A. B1B достигнет ее через некоторое время, которое можно определить отношением:

Δt=CBv1

В момент, когда вторичная волна в точке B только начинает возбуждаться, волна от точки A уже имеет вид полусферы, радиус которой определяется выражением:

AD=v2Δt

Волновую поверхность преломленной волны можно получить, проведя поверхность, касательную всем вторичным волнам во второй среде, центры которых лежат на границе раздела сред. В данном случае, ею является плоскость BD. Она является огибающей вторичных волн.

Угол падения α равен CAB в треугольнике ABC (стороны одного из этих углов перпендикулярны сторонам другого). Следовательно:

CB=v1Δt=ABsinα

Угол преломления β равен углу ABD в треугольнике ABD. Поэтому:

AD=v2Δt=ABsinβ

Поделим первое выражение на второе и получим:

sinαsinβ=v1v2=n

Закон преломления света

Падающий луч, луч преломленный и перпендикуляр, восстановленный в точке падения, лежат в одной плоскости. Отношение синуса угла падения к синусу угла преломления есть величина постоянная для двух сред.

Пример №3. Угол падения параллельных лучей на плоскопараллельную пластинку равен 60о. Найдите расстояние между точками, в которых из пластины выходят параллельные лучи, если расстояние между лучами, прошедшими сквозь пластину, равно 0,7 м.

Сначала построим рисунок хода лучей до пластины, внутри нее и после нее. Расстояние между лучами, прошедшими сквозь пластину, обозначим за l. Оно равна длине перпендикуляра, соединяющего эти лучи.

Значение величины угла β, который составляет нормаль к пластине и направлением распространения луча в ней, определяется законом преломления света:

sinαsinβ=n

Луч выходит из пластины под некоторым углом γ таким, что:

sinβsinγ=1n

Следовательно:

n=sinγsinβ=sinαsinβ

Отсюда: sinγ=sinα или γ= α. Если вспомнить геометрические законы, можно сделать вывод, что расстояние между пластинами, являющееся гипотенузой прямоугольного треугольника, можно вычислить путем деления катета на косинус угла между ним и гипотенузой:

L=lcos60°=0,70,5=1,4 (м)

Величина n — относительный показатель преломления.

Физический смысл показателя преломления заключается в том, что он равен отношению скоростей света в средах, на границе между которыми происходит преломление.

n=v1v2

Различают также абсолютный показатель преломления — показатель преломления среды относительно вакуума. Он равен синусу угла падения к синусу угла преломления при переходе светового луча из вакуума в данную среду.

Поскольку в вакууме скорость света максимальна, абсолютный показатель преломления можно выразить формулой:

n=cv1

где v1 — скорость света в среде, c — скорость света в вакууме.

Между абсолютными и относительными показателями преломления есть взаимосвязь. Пусть скорость распространения света в первой среде равна v1, во второй — v2. Тогда абсолютные показатели преломления для первой и второй среды равны:

n1=cv1

n2=cv2

Тогда относительный показатель преломления при переходе света из первой среды во вторую будет равен отношению абсолютного показателя преломления второй среды к абсолютному показателю преломления первой среды:

n=v1v2=n2n1

Внимание!

Среду с меньшим абсолютным показателем преломления принято называть оптически менее плотной средой, а среду с большим абсолютным показателем преломления — оптически более плотной.

Пример №4. Определить показатель преломления воды относительно алмаза.

n=nвnа

Абсолютные показатели преломления воды и алмаза — постоянные табличные величины.

n=1,332,420,55

Полное отражение

Закон преломления света позволяет объяснить интересное и практически важное явление — полное отражение света.

При прохождении света из оптически менее плотной среды в более плотную, к примеру, из воздуха в стекло или воду, v1>v2. Следовательно, согласно закону преломления показатель преломления n > 1. Поэтому α > β (см. рисунок а). В результате преломления луч приближается к перпендикуляру, восстановленному к точке падения луча.

Если же направить луч света в обратном направлении — из оптически более плотной среды в оптически менее плотную вдоль ранее преломленного луча (см. рисунок б), то закон преломления запишется следующим образом:

sinαsinβ=v2v1=1n

Преломленный луч по выходе из оптически более плотной среды будет направлен по линии ранее падавшего луча, поэтому α < β, т. е. преломленный луч в этом случае отдаляется от перпендикуляра, восстановленного в точке падения к границе раздела сред. По мере увеличения угла α угол преломления β также увеличивается. При этом, согласно закону преломления света, он всегда будет больше угла α. Наконец, при некотором угле падения α значение угла преломления β приблизится к 90°, и преломленный луч будет направлен почти по границе раздела двух сред (см. рисунок в). Наибольшему возможному углу преломления β = 90° соответствует угол падения α0.

Попробуем выяснить, что произойдет при α > α0. При падении света на границу двух сред световой луч, как мы уже говорили ранее, частично отражается и частично преломляется. Но при α > α0 преломление света невозможно. Значит, луч должен полностью отразиться. Это явление и называется полным отражением света.

Примеры полного отражения света:

  • блеск от ограненного алмаза;
  • блеск капель росы на солнце;
  • внутреннее отражение предметов, находящихся под водой.

Определение

Угол полного отражения — угол падения α0, соответствующий углу преломления 90°.

При sin β = 1 (что соответствует углу 90°) угол полного отражения можно определить по формуле:

sinα0=1n

Пример №5. Луч света, идущий из толщи воды, полностью отражается от ее поверхности. Выйдет ли луч в воздух, если на поверхность воды налить слой кедрового масла?

Синус угла полного отражения для луча, идущего из воды к воздуху:

sinα0=1n1

sinα0 n1=1

где n1 — показатель преломления воды.

Запишем закон преломления света для случая, когда на поверхность воды налито масло:

Тогда синус угла полного отражения для луча, идущего из воды к маслу:

sinα0sinβ=n2n1

где n2 — показатель преломления масла.

Тогда:

sinβ=1n2

Эта формула соответствует случаю, когда угол β является углом полного отражения. Следовательно, луч света за пределы масляной пленки в воздух не выйдет.

Практическое применение явления полного отражения света

Явление полного отражения света применяют в волоконной оптике для передачи света и изображения по пучкам прозрачных гибких волокон — световодов. Световод — это стеклянное волокно цилиндрической формы, покрытое оболочкой из прозрачного материала с меньшим, чем у волокна, показателем преломления.

За счет многократного полного отражения свет может быть направлен, либо по прямому, либо по изогнутому пути (см. рисунок слева). Волокна собираются в жгуты. При этом по каждому из волокон передается какой-нибудь элемент изображения (см. рисунок справа). Жгуты из волокон используются, например, в медицине для исследования внутренних органов.

В последнее время волоконная оптика широко используется для быстрой передачи компьютерных сигналов. По волоконному кабелю передается модулированное лазерное излучение.

Задание EF17610

Ученик провёл опыт по преломлению монохроматического света, представленный на фотографии.

Затем вся установка была помещена в воду. Как изменятся частота световой волны, длина волны, падающей на стекло, и угол преломления?

Для каждой величины определите соответствующий характер изменения:


Запишите в таблицу выбранные цифры для каждого ответа. Цифры в ответе могут повторяться.


Алгоритм решения

1.Описать эксперимент, проведенный учеником.

2.Установить, как изменяется частота световой волны при перемещении установки из воздуха в воду.

3.Установить, как при этом изменяется длина световой волны.

4.Установить, как при этом изменяется угол преломления.

Решение

Ученик направил луч монохроматического света на стекло под углом 30 градусов к нормали. При этом луч вышел под углом 20 градусов. Это говорит о том, что свет из менее плотной оптической среды попал в более плотную.

Частота световой волны — характеристика, не зависящая от условий распространения этой волны. Поэтому при перемещении установки из воздуха в воду частота останется прежней.

Чтобы установить, как меняется длина световой волны и угол преломления. Нужно рассчитать изменение показателя преломления света. Относительный показатель преломления в первом и втором опыте будет соответственно равен:

sinαsinβ=nвоздухстекло

sinαsinγ=nводастекло

Относительные показатели преломления можем выразить через абсолютные:

nвоздухстекло=nстеклоnвоздух

nводастекло=nстеклоnвода

Абсолютный показатель преломления — табличная величина. Мы возьмем приблизительный значения: для воздуха — 1, для воды — 1,33, для стекла — 1,5. В действительности абсолютный показатель преломления стекла может составлять от 1,43 до 2,17. Но это не столь важно, поскольку важно лишь то, что он в любом случае больше абсолютного показателя преломления воды.

Получим:

nвоздухстекло=1,51=1,5

nводастекло=1,51,331,3

Видно, что при перемещении из воздуха показатель преломления уменьшился. Тогда:

sinαsinγ=1,3

Так как числитель в левой части уравнения остался прежним, а число в правой части уменьшилось, то синус угла преломления увеличился. Поскольку синус угла находится в прямой зависимости от величины угла, то и угол преломления увеличился.

Длина волны определяется формулой:

λ=vν

Учтем, что скорость распространения света в более плотной среде уменьшается. Если скорость уменьшилась, то длина воды тоже уменьшилась, поскольку между ними существует прямо пропорциональная зависимость.

Ответ: 321

pазбирался: Алиса Никитина | обсудить разбор

Задание EF18593

Свет падает на горизонтальное плоское зеркало. Угол между падающим и отражённым лучами равен 60°. Каким станет угол между этими лучами, если повернуть зеркало на 20°, как показано на рисунке?


Алгоритм решения

1.Записать известные данные.

2.Зарисовать рисунок после поворота зеркала.

3.Представить решение задачи в общем виде.

4.Подставить неизвестные данные и вычислить искомую величину.

Решение

Запишем исходные данные:

 Угол между падающим и отраженным углом: γ1 = 60о.

 Угол поворота угла: φ = 20о.

Построим рисунок с учетом того, что зеркало повернули:

Поскольку угол падения, равен углу отражения, то:

α1+β1=60°

α1=β1

2α1=60°

α1=60°2=30°

На рисунке видно, что после переворачивания зеркала угол падения α увеличился на угол переворота:

α=α1+φ=30°+20°=50°

Так как угол падения равен углу отражения, то:

α=β=50°

Отсюда угол между лучом падающим и лучом отраженным равен:

γ=α+β=50°+50°=100°

Ответ: 100

pазбирался: Алиса Никитина | обсудить разбор

Задание EF19015

На дне бассейна с водой находится небольшая лампочка. На поверхности воды плавает круглый плот – так, что центр плота находится точно над лампочкой. Определите глубину бассейна Н, если минимальный радиус плота, при котором свет от лампочки не выходит из воды, R = 2,4 м. Сделайте рисунок, поясняющий решение. Толщиной плота пренебречь. Показатель преломления воды n = 4/3.


Алгоритм решения

1.Записать исходные данные.

3.Записать закон полного отражения.

4.Выполнить решение в общем виде.

5.Подставить известные данные и вычислить искомую величину.

Решение

Запишем исходные данные:

 Радиус круглого плота: R = 2,4 м.

 Показатель преломления воды: n = 4/3.

Выполним рисунок. Проведем перпендикуляры к поверхности: перпендикуляр от точечного источника света, а также нормали, проведенные через края плота.

Чтобы свет лампочки не выходил из воды, лучи света от лампочки, направленные к границе между краем плота и поверхностью воды, должны полностью отражаться. Это возможно только при выполнении следующего условия:

sinα=1n

Поскольку вершина S треугольника ABS лежит строго под центром круглого плота, этот треугольник является равнобедренным. Причем перпендикуляр, восстановленный к основанию треугольника ABSO — делит это основание на 2 равные стороны. Одновременно он делит угол S этого треугольника на 2 равные части, так как он является одновременно перпендикуляром, медианой и биссектрисой.

Пусть α — угол падения луча. Тогда угол OSB будет равен этому углу как накрест лежащие углы.

Треугольник OSB — прямоугольный. Причем искомая величина — глубина бассейна — является одним из его катетов. Из курса геометрии известно, что катет равен произведения второго катета на котангенс прилежащего угла. Второй катет в нашем случае — радиус круглого плота. Прилежащий угол равен углу падения. Следовательно:

H=Rcotα

Котангенс угла определяется как отношение косинуса этого угла к его синусу:

cotα=cosαsinα

Косинус угла можем выразить из основного тригонометрического тождества:

sin2α+cos2α=1

Следовательно:

cosα=1sin2α

Отсюда котангенс равен:

cotα=1sin2αsinα

Тогда глубина бассейна:

H=Rcotα=R1sin2αsinα

Из закона полного отражения вспомним, что синус угла падения есть величина, обратная показателю преломления воды. Тогда эта формула примет вид:

H=R1(1n)21n=Rn11n2

Подставим известные данные и получим:

H=2,4·4311(43)2=3,21916=3,2740,8·2,65=2,12 м

Ответ: 2,12

pазбирался: Алиса Никитина | обсудить разбор

Задание EF17706

Стеклянную линзу (показатель преломления стекла nстекла = 1,54), показанную на рисунке, перенесли из воздуха (nвоздуха = 1) в воду (nводы = 1,33). Как изменились при этом фокусное расстояние и оптическая сила линзы?

Ответ:

а) Фокусное расстояние уменьшилось, оптическая сила увеличилась.

б) Фокусное расстояние увеличилось, оптическая сила уменьшилась.

в) Фокусное расстояние и оптическая сила увеличились.

г) Фокусное расстояние и оптическая сила уменьшились.


Алгоритм решения

1.Установить характер преломления лучей линзой при ее перемещении из воздуха в воду.

2.Выяснить, как от этого зависят фокусное расстояние и оптическая сила линзы.

Решение

Чтобы узнать, что произойдет с лучами света при прохождении их сквозь линзу, погруженную воду, найдем относительные показатели преломления:

nвоздухстекло=nстеклоnвоздух=1,541=1,54

nводастекло=nстеклоnвода=1,541,331,16

Видно, что относительный показатель преломления уменьшился. Значит, преломленный линзой луч будет менее отклоняться от нормали, проведенной в точке падения на линзу. Следовательно, чтобы достигнуть главной оптической оси, ему придется пройти большее расстояние. Это говорит о том, что фокусное расстояние линзы увеличится.

Оптическая сила линзы — величина, обратная ее фокусному расстоянию. Если оно увеличится, то оптическая сила уменьшится.

Ответ: б

pазбирался: Алиса Никитина | обсудить разбор

Алиса Никитина | Просмотров: 7.2k

Преломление
света
 —
явление, при котором луч света, переходя
из одной среды в другую, изменяет
направление на границе этих сред.

Преломление
света происходит по следующему
закону:
Падающий
и преломленный лучи и перпендикуляр,
проведенный к границе раздела двух сред
в точке падения луча, лежат в одной
плоскости. Отношение синуса угла падения
к синусу угла преломления есть величина
постоянная для двух сред:

,
гдеα — угол
падения,

β — угол
преломления,

n — постоянная
величина, не зависящая от угла падения.

При
изменении угла падения изменяется и
угол преломления. Чем
больше угол падения, тем больше угол
преломления.

Если
свет идет из среды оптически менее
плотной в более плотную среду, то угол
преломления всегда меньше угла падения: β
< α.

Луч
света, направленный перпендикулярно к
границе раздела двух сред, проходит из
одной среды в другую без
преломления.

абсолютный
показатель преломления вещества —
величина, равная отношению фазовых
скоростей света (электромагнитных волн)
в вакууме и в данной среде n=c/v 
Величина
n, входящая в закон преломления, называется
относительным показателем преломления
для пары сред. 

Величина
n есть относительный показатель
преломления среды В по отношению к среде
А, а n’ = 1/n есть относительный показатель
преломления среды А по отношению к среде
В. 

Эта
величина при прочих равных условиях
больше единицы при переходе луча из
среды более плотной в среду менее
плотную, и меньше единицы при переходе
луча из среды менее плотной в среду
более плотную (например, из газа или из
вакуума в жидкость или твердое тело).
Есть исключения из этого правила, и
потому принято называть среду оптически
более или менее плотной, чем другая. 

Луч,
падающий из безвоздушного пространства
на поверхность какой-нибудь среды В,
преломляется сильнее, чем при падении
на нее из другой среды А; показатель
преломления луча, падающего на среду
из безвоздушного пространства, называется
его абсолютным показателем преломления
.

(Абсолютный
— относительно вакуума. 
Относительный
— относительно любого другого вещества
(того же воздуха, например). 
Относительный
показатель двух веществ есть отношение
их абсолютных показателей. )

Полное
внутреннее отражение
 —
внутреннее отражение, при условии, что
угол падения превосходит некоторый
критический угол. При этом падающая
волна отражается полностью, и
значение коэффициента
отражения превосходит
его самые большие значения для полированных
поверхностей. Коэффициент отражения
при полном внутреннем отражении не
зависит от длины
волны.

В
оптике это явление наблюдается
для широкого спектра электромагнитного
излучения,
включая рентгеновский
диапазон.

В
геометрической оптике явление объясняется
в рамках закона
Снелла.
Учитывая, что угол преломления не может
превышать 90°, получаем, что при угле
падения, синус которого
больше отношения меньшего показателя
преломления к большему показателю,
электромагнитная волна должна полностью
отражаться в первую среду.

В
соответствии с волновой теорией явления,
электромагнитная волна всё же проникает
во вторую среду — там распространяется
так называемая «неоднородная волна»,
которая экспоненциально затухает
и энергию с собой не уносит. Характерная
глубина проникновения неоднородной
волны во вторую среду порядка длины
волны.

Соседние файлы в предмете [НЕСОРТИРОВАННОЕ]

  • #
  • #
  • #
  • #
  • #
  • #
  • #
  • #
  • #
  • #
  • #

Понравилась статья? Поделить с друзьями:
  • Как найти покупателей на нефтепродукты
  • Как найти легендарные импланты в киберпанке
  • Как найти совершенную газом работу 300 дж
  • Как составить план классного мероприятия
  • Как найти свой уникальный продукт